The question bank may take some time to load… Just enough time to stretch, blink a few times, and question your life choices — but not too long, we promise!
We recommend going Full Screen for the best experience. Have Fun !
Report a question
CVS – 2022
Questions from CVS’s 2022 Module + Annual Exam
“Which coronary artery supplies the bottom (inferior) part of the heart, and which ECG leads correspond to that region?”
1 / 101
Category:
CVS – Physiology
A 55-year-old man presents to the emergency room with a complaint of sudden chest pain. His electrocardiogram reveals an ST-segment elevation in leads II, III, and avF. Laboratory investigation of his blood sample for the presence of cardiac enzymes shows raised levels of troponin T and a diagnosis of myocardial infarction is made. Which of the following walls of the myocardium is infarcted?
Understanding ECG Lead Placement and Myocardial Infarction Location
ST-segment elevation in specific leads on an ECG correlates with the region of infarction and the occluded coronary artery .
Lead Changes and Infarct Location:
Leads with ST Elevation
Affected Myocardial Wall
Likely Artery Involved
II, III, aVF
Inferior wall
Right Coronary Artery (RCA)
V1–V4
Anterior wall
Left Anterior Descending (LAD) artery
I, aVL, V5, V6
Lateral wall
Left Circumflex (LCX) artery
V7–V9
Posterior wall
Posterior Descending Artery (PDA) (branch of RCA or LCX)
V1–V6, I, aVL
Anterolateral wall
LAD or LCX
Why Is the Inferior Wall Infarcted?
ST elevation in leads II, III, and aVF indicates damage to the inferior wall of the left ventricle .
The right coronary artery (RCA) supplies the inferior part of the heart in most individuals.
Inferior myocardial infarction (MI) is often caused by RCA occlusion.
Breakdown of Incorrect Options:
Anterior Wall → Incorrect
The anterior wall is supplied by the left anterior descending (LAD) artery .
Anterior MI is seen with ST elevation in leads V1–V4 , which is not present in this patient.
Lateral Wall → Incorrect
The lateral wall is supplied by the left circumflex (LCX) artery .
Lateral MI presents with ST elevation in leads I, aVL, V5, and V6 , which is not the case here.
Posterior Wall → Incorrect
Posterior MI is usually due to occlusion of the PDA (a branch of the RCA or LCX) .
It presents with ST depression in V1–V3 with ST elevation in posterior leads (V7–V9) .
Anterolateral Wall → Incorrect
Anterolateral infarction involves the LAD or LCX , leading to ST elevation in V1–V6, I, and aVL .
This patient has ST elevation in II, III, and aVF , which corresponds to inferior infarction, not anterolateral.
Myocardial infarction (MI) is strongly associated with modifiable and non-modifiable risk factors . Non-modifiable factors cannot be changed, but modifiable factors can be controlled or eliminated to reduce MI risk. Among the options, which modifiable factor has the greatest impact on reducing cardiovascular disease risk?
2 / 101
Category:
CVS – Community Medicine/ Behavioural Sciences
A 50-year-old smoker develops chest pain while doing an activity. What is the most important modifiable factor to reduce the risk of myocardial infarction?
The most important modifiable risk factor to reduce the risk of myocardial infarction (MI) is smoking cessation .
Why “Smoking” is the Correct Answer:
Smoking is one of the strongest risk factors for MI due to its impact on:
Endothelial dysfunction → Damage to blood vessel walls.
Increased LDL (“bad cholesterol”) and decreased HDL (“good cholesterol”) → Promotes atherosclerosis.
Increased platelet aggregation and thrombosis → Higher risk of clot formation leading to MI.
Vasoconstriction and reduced oxygen supply → Nicotine increases blood pressure and heart rate.
Quitting smoking significantly reduces MI risk within months and brings the risk down to near non-smoker levels within a few years .
Epidemiological studies show that smoking cessation is the single most impactful lifestyle change in preventing cardiovascular disease.
Why the Other Options Are Incorrect:
Gender – Incorrect
Gender is a non-modifiable risk factor .
While men have a higher risk of MI at a younger age , women’s risk increases after menopause .
However, gender cannot be changed , making it irrelevant for risk modification .
Physical Inactivity – Incorrect (Important but Not the Most Impactful)
Exercise improves cardiovascular health and reduces MI risk .
However, while being active lowers risk, smoking cessation has a larger immediate impact.
A smoker who exercises still has a higher MI risk than a non-smoker who is inactive .
Age – Incorrect
Age is a non-modifiable factor .
MI risk increases with age , but we cannot change aging .
Race – Incorrect
Certain racial groups have different predispositions (e.g., African Americans have higher hypertension rates), but race is not modifiable .
Controlling modifiable risk factors (like smoking) is more important than genetic predisposition.
“If one road is blocked, how does traffic still reach its destination? The body has a backup route that forms over time to bypass the obstruction.”
3 / 101
Category:
CVS – Physiology
During coronary angiography, a male patient was found to have a blockage of the branch of a coronary artery. However, the patient did not develop any symptoms of myocardial infarction. What could be the possible reason for this?
Collateral circulation refers to the development of alternative blood flow pathways that help maintain perfusion to a region of the heart even when a major coronary artery is blocked. This explains why some patients with coronary artery disease (CAD) may have significant arterial blockages but do not experience symptoms of myocardial infarction (MI) .
How Collateral Circulation Works:
Chronic gradual occlusion of a coronary artery stimulates the development of small anastomotic connections between adjacent blood vessels.
Shear stress and hypoxia trigger the release of vascular endothelial growth factors (VEGF) and angiogenesis , leading to the formation of new collateral blood vessels .
These collateral vessels compensate for the restricted blood flow, allowing oxygen delivery to the myocardium and preventing infarction.
Why the Other Options Are Incorrect:
Vasodilation of the Blocked Artery (Incorrect)
Vasodilation cannot clear a complete or significant blockage .
It only improves blood flow in partially occluded vessels , but a fully blocked artery needs collateral circulation.
Increased Compliance of the Blocked Artery (Incorrect)
Arterial compliance refers to the elasticity of blood vessels .
Increased compliance does not restore blood flow in a blocked artery ; it only affects pressure regulation.
Bilateral Circulation (Incorrect)
The coronary circulation is not bilateral like the systemic circulation (e.g., brain’s Circle of Willis).
Each coronary artery has specific areas it supplies, and there is no direct bilateral compensation .
Thrombolysis (Incorrect)
Thrombolysis (clot breakdown) occurs spontaneously or via medical intervention (e.g., tPA, streptokinase) .
If the patient had spontaneous thrombolysis , there would have been previous ischemic symptoms before the blockage was cleared.
In this case, the patient was asymptomatic , suggesting that pre-existing collateral circulation was responsible.
Clinical Correlation:
Patients with slow, chronic coronary artery disease (e.g., stable angina) often develop collateral circulation , reducing the risk of an acute MI.
Acute blockages (e.g., plaque rupture) in patients without collateral circulation lead to severe ischemia and myocardial infarction .
“Which apolipoprotein is most closely linked to HDL function and is required for the enzyme that esterifies cholesterol?”
4 / 101
Category:
CVS – BioChemistry
A 50-year-old woman is diagnosed with familial LCAT (lecithin-cholesterol acyltransferase) deficiency after a renal biopsy. She has a marked decrease in esterified cholesterol and low levels of high-density lipoproteins (HDL), as well as decreased activity of LCAT. Which of the following is a principal activator of LCAT?
Understanding LCAT and Its Activation
Lecithin-cholesterol acyltransferase (LCAT) is an enzyme that plays a key role in cholesterol metabolism by catalyzing the esterification of free cholesterol in plasma. This allows cholesterol to be efficiently carried by HDL particles and transported back to the liver for excretion (reverse cholesterol transport ).
Role of Apolipoprotein A-1 (ApoA-1) in LCAT Activation:
Apolipoprotein A-1 (ApoA-1) is the principal activator of LCAT .
ApoA-1 is primarily associated with HDL , making it essential for cholesterol esterification and HDL maturation .
In LCAT deficiency , cholesterol remains in its free form , leading to:
Low HDL levels (due to impaired maturation).
Accumulation of free cholesterol in tissues (causing kidney disease, corneal opacities, and anemia).
Breakdown of Incorrect Options:
Apolipoprotein C-2 → Incorrect
ApoC-2 is an activator of lipoprotein lipase (LPL) , which breaks down triglycerides in chylomicrons and VLDL.
It has no role in LCAT activation .
Apolipoprotein B → Incorrect
ApoB is involved in lipoprotein structural integrity :
ApoB-100 → Found in VLDL, IDL, and LDL , important for LDL receptor binding .
ApoB-48 → Found in chylomicrons , important for intestinal lipid absorption.
Neither form of ApoB activates LCAT.
Apolipoprotein E → Incorrect
ApoE is essential for remnant uptake by the liver (chylomicron remnants, IDL).
It plays no role in LCAT activation .
Apolipoprotein B-48 → Incorrect
ApoB-48 is involved in chylomicron assembly and secretion .
It does not activate LCAT .
Consider the balance between moderate and vigorous physical activity in public health recommendations. What is the minimum duration and frequency recommended by major health organizations, such as the American Heart Association (AHA) and WHO , to significantly reduce cardiovascular disease risk?
5 / 101
Category:
CVS – Community Medicine/ Behavioural Sciences
In healthy individuals, physical activity reduces the risk of cardiovascular disease mortality by 20-30%. What are the current guidelines regarding the frequency of physical activity?
Current physical activity guidelines recommend a combination of moderate and vigorous exercise to achieve cardiovascular health benefits. The recommended amount of moderate-intensity aerobic exercise for adults is at least 150 minutes per week , which translates to 30 minutes of moderate exercise, five days a week .
Why “Thirty minutes of moderate exercise five days a week” is the Correct Answer:
American Heart Association (AHA) & World Health Organization (WHO) Guidelines:
Adults should engage in at least 150 minutes of moderate-intensity aerobic exercise per week (or 75 minutes of vigorous-intensity exercise per week ).
This can be achieved by 30 minutes of moderate-intensity exercise, five days a week .
Alternatively, individuals can opt for 25 minutes of vigorous exercise, three days a week or an equivalent combination of moderate and vigorous activity.
Examples of Moderate Exercise:
Brisk walking
Cycling at a moderate pace
Water aerobics
Light jogging
This level of physical activity helps reduce cardiovascular disease mortality, improve blood pressure, lower cholesterol, and enhance overall heart health .
The location of the stab wound is crucial in determining which structure is most likely to be injured. The right intercostal space suggests what?
6 / 101
Category:
CVS – Anatomy
A stab wound in the right intercostal space is most likely to injure which one of the following structures?
Why “Right Atrium” is the Correct Answer?
The right atrium is located on the right side of the sternum , extending from the right 3rd to 5th intercostal spaces .
It receives venous blood from the superior and inferior vena cava and is positioned more laterally than the right ventricle.
A stab wound in the right intercostal space (especially in the lower right parasternal region) is more likely to penetrate the right atrium before reaching the right ventricle .
The right atrial wall is relatively thin , making it more prone to perforation compared to the right ventricle.
Why the Other Options Are Incorrect?
Apex of Heart – Incorrect
The apex of the heart is formed by the left ventricle and is located in the left 5th intercostal space .
A right-sided stab wound would not reach the apex .
Right Ventricle – Incorrect
The right ventricle is the most anterior heart chamber , but it is slightly left of the midline .
A right-sided intercostal wound is more likely to hit the right atrium before reaching the right ventricle .
Liver – Incorrect
The liver is located in the right upper quadrant but below the diaphragm .
A low chest wound near the costal margin could injure the liver, but not a wound in the upper right intercostal space .
Pulmonary Artery – Incorrect
The pulmonary artery is positioned more superiorly , emerging from the right ventricle.
It is not directly in the path of a right intercostal stab wound .
“Which congenital defect results from improper development of the structures related to the heart, specifically those affected the AV valves.”
7 / 101
Category:
CVS – Pathology
Which of the following congenital defects is marked by the incomplete fusion of the endocardial tissue?
Understanding Atrioventricular Septal Defect (AVSD) and Endocardial Cushion Defects
The endocardial cushions are critical structures in the embryonic heart that contribute to the formation of the atrial septum, ventricular septum, and the atrioventricular (AV) valves (mitral and tricuspid valves).
Incomplete fusion of the endocardial cushions leads to an atrioventricular septal defect (AVSD) , also known as an atrioventricular canal defect .
This defect results in abnormal communication between the atria and ventricles due to incomplete formation of the AV septum and maldevelopment of the AV valves .
Types of AVSD:
Complete AVSD:
Large common AV valve instead of separate mitral and tricuspid valves.
Severe mixing of oxygenated and deoxygenated blood.
Strongly associated with Down syndrome (Trisomy 21).
Partial AVSD:
Involves defects in the lower atrial septum (primum ASD) and/or abnormal AV valves .
Less severe than complete AVSD , but still leads to left-to-right shunting of blood.
Breakdown of Incorrect Options:
Transposition of Great Vessels → Incorrect
This results from failure of the aorticopulmonary septum to spiral correctly , leading to the aorta arising from the right ventricle and the pulmonary artery from the left ventricle .
It is not related to endocardial cushion defects .
Tetralogy of Fallot (TOF) → Incorrect
TOF is caused by abnormal development of the conotruncal septum , leading to:
Pulmonary stenosis
Right ventricular hypertrophy
Overriding aorta
Ventricular septal defect (VSD)
It is not due to endocardial cushion defects .
Coarctation of the Aorta → Incorrect
This is a narrowing of the aortic arch , often near the ductus arteriosus.
It is not related to endocardial cushion development .
Pulmonary Stenosis → Incorrect
This refers to narrowing of the pulmonary valve or outflow tract , typically due to abnormal conotruncal development .
It does not involve the endocardial cushions .
Consider the major risk factors for vascular disease in this patient: hypertension, smoking, and dyslipidemia . What chronic vascular condition results?
8 / 101
Category:
CVS – Pathology
A 58-year-old man presents to the outpatient department with a complaint of reduced exercise tolerance over the past five years. In the past year, he has noticed chest pain after climbing stairs. He smokes two packs of cigarettes per day. On examination, he has a blood pressure of 155/95 mmHg. Laboratory investigations include total serum cholesterol of 245 mg/dL, with high-density lipoprotein cholesterol of 22 mg/dL. Which of the following vascular abnormalities is most likely to have developed in this patient?
This 58-year-old male presents with progressive exercise intolerance and chest pain with exertion , which suggests coronary artery disease (CAD) . His risk factors include:
Smoking (2 packs/day) → Causes endothelial damage and promotes plaque formation.
Hypertension (155/95 mmHg) → Contributes to arterial wall stress and damage.
Dyslipidemia (High total cholesterol: 245 mg/dL, Low HDL: 22 mg/dL) → Facilitates plaque buildup.
Given this clinical picture, the most likely vascular abnormality is atherosclerosis , a chronic disease characterized by lipid accumulation, inflammation, and progressive narrowing of the arteries.
Why “Atherosclerosis” is the Correct Answer:
Atherosclerosis is the leading cause of coronary artery disease (CAD), peripheral artery disease (PAD), and cerebrovascular disease (stroke).
It is characterized by intimal plaques composed of lipids, inflammatory cells, smooth muscle proliferation, and extracellular matrix deposition .
Plaque formation narrows the arterial lumen , reducing blood flow and leading to ischemic symptoms (such as exertional chest pain).
Risk factors for atherosclerosis include hypertension, smoking, and hyperlipidemia , all of which are present in this patient.
Why the Other Options Are Incorrect:
Cardiac Hypertrophy – Incorrect
While hypertension can lead to left ventricular hypertrophy (LVH) , it is not a vascular abnormality but rather a myocardial adaptation to increased afterload.
LVH can contribute to heart failure and arrhythmias , but it does not explain the primary vascular pathology in this case .
Medial Calcific Sclerosis – Incorrect
Medial calcific sclerosis (Monckeberg sclerosis) involves calcium deposits in the media of medium-sized arteries but does not cause luminal narrowing or ischemia .
It is usually asymptomatic and an incidental finding on imaging.
This patient’s symptoms are due to atherosclerotic narrowing, not medial calcification .
Hyperplastic Arteriosclerosis – Incorrect
Hyperplastic arteriolosclerosis (“onion-skin” thickening) occurs in malignant hypertension (BP often >180/120 mmHg), leading to acute end-organ damage .
This patient has chronic hypertension , but not at a level that suggests malignant hypertension.
The clinical scenario (chest pain on exertion) is more consistent with atherosclerosis affecting the coronary arteries .
Deep Venous Thrombosis (DVT) – Incorrect
DVT occurs due to venous stasis, hypercoagulability, and endothelial injury (Virchow’s triad) .
This patient’s symptoms point to arterial disease (atherosclerosis) rather than venous thrombosis.
DVT typically causes leg swelling, pain, and risk of pulmonary embolism , not exertional chest pain.
“Why does one ventricle need to work harder than the other, even though both pump the same amount of blood per beat?”
9 / 101
Category:
CVS – Physiology
Which of the following reasons accounts for a thicker left ventricular wall as compared to the right ventricular wall?
Understanding Left Ventricular Wall Thickness
The left ventricle has a thicker wall than the right ventricle because it needs to generate greater pressure to pump blood throughout the systemic circulation , which has higher resistance compared to the pulmonary circulation.
Key Reasons for the Thick Left Ventricular Wall:
Systemic vs. Pulmonary Circulation:
The left ventricle pumps blood to the entire body (systemic circulation) , requiring much higher pressure (~120 mmHg in systole) .
The right ventricle pumps blood to the lungs (pulmonary circulation) , which has lower resistance (~25 mmHg in systole) .
Because of this, the left ventricular myocardium must be thicker and stronger to generate the higher force needed to overcome systemic vascular resistance (SVR).
Pressure Generation, Not Volume Handling:
Both ventricles pump approximately the same volume of blood per beat (stroke volume) , but the left ventricle must do so against much greater resistance.
The higher workload leads to increased myocardial thickness over time (physiological hypertrophy ).
Breakdown of Incorrect Options:
“To pump blood through a smaller valve” → Incorrect
The aortic valve is not significantly smaller than the pulmonary valve in a way that would require a thicker wall.
The primary factor is pressure, not valve size .
“To pump a greater volume of blood” → Incorrect
The left and right ventricles pump equal volumes of blood per beat .
The key difference is pressure generation, not volume handling .
“To accommodate a greater volume of blood” → Incorrect
The left ventricle does not accommodate more blood than the right ventricle .
The end-diastolic volume (EDV) is similar in both ventricles.
“To expand the thoracic cage during diastole” → Incorrect
The left ventricle does not contribute to thoracic expansion .
Diaphragm movement and respiratory muscles control thoracic expansion.
Think about how tissues get energy from the fat in circulating chylomicrons. Which enzyme would be necessary to break down the triglycerides inside these lipoproteins so that fatty acids can be delivered for energy or storage?”
10 / 101
Category:
CVS – BioChemistry
Chylomicrons are transformed into chylomicron remnants by the hydrolysis of the core triglycerides through the action of which enzyme?
Chylomicrons are lipoproteins responsible for transporting dietary triglycerides (TGs) and cholesterol from the intestines to peripheral tissues via the lymphatic and circulatory systems.
Once in the bloodstream, chylomicrons interact with Lipoprotein Lipase (LPL) , an enzyme anchored to capillary endothelial cells in muscle (especially skeletal and cardiac) and adipose tissue .
LPL hydrolyzes the triglycerides present in the core of chylomicrons into free fatty acids (FFAs) and glycerol .
The FFAs are taken up by tissues for energy production (muscle) or storage (adipose tissue) .
After significant triglyceride removal, chylomicrons shrink in size and become “chylomicron remnants” , which are richer in cholesterol esters .
These remnants are then cleared by the liver via ApoE-mediated receptor uptake .
Thus, Lipoprotein Lipase (LPL) is the key enzyme in transforming chylomicrons into chylomicron remnants by hydrolyzing their triglycerides.
Why the Other Options Are Incorrect:
Chylomicron transferase – Incorrect
No such enzyme exists in lipid metabolism.
The term may resemble Microsomal Triglyceride Transfer Protein (MTP) , which assists in assembling chylomicrons in enterocytes but does not hydrolyze their triglycerides.
Cholesteryl esterase – Incorrect
This enzyme hydrolyzes cholesteryl esters into free cholesterol and fatty acids .
It is mainly found in the liver and lysosomes , not the bloodstream.
It plays a role in cholesterol metabolism but does not degrade chylomicron triglycerides .
Apolipoprotein synthase – Incorrect
There is no enzyme called Apolipoprotein Synthase .
Apolipoproteins (such as ApoB-48 in chylomicrons) are produced in the intestines and liver , but their synthesis is unrelated to triglyceride hydrolysis.
Cholesteryl transferase – Incorrect
This could refer to Cholesteryl Ester Transfer Protein (CETP) , which facilitates the transfer of cholesteryl esters between lipoproteins .
CETP is involved in HDL metabolism , not in chylomicron breakdown.
It does not hydrolyze triglycerides .
“Once the ventricles receive the electrical impulse, they contract, but they must build up enough pressure to push blood into the arteries. What happens in the very first moments of contraction?”
11 / 101
Category:
CVS – Physiology
Which of the following occurs immediately after the QRS complex?
The QRS complex on an ECG (electrocardiogram) represents ventricular depolarization , which precedes ventricular contraction . Immediately after the QRS complex, the ventricles begin to contract , marking the isovolumic contraction phase of the cardiac cycle.
Step-by-Step Breakdown of Events:
QRS Complex (Ventricular Depolarization)
Electrical activation of the ventricles.
Triggers the beginning of ventricular systole .
Isovolumic Contraction (Correct Answer)
Occurs immediately after the QRS complex .
The ventricles contract, but the semilunar valves (aortic & pulmonary) remain closed because ventricular pressure has not yet exceeded arterial pressure.
This phase builds up pressure in the ventricles before blood is ejected.
Ventricular Systole (Incorrect)
Ventricular systole begins with isovolumic contraction but continues into the ejection phase when the semilunar valves open.
The actual ejection of blood occurs slightly after the QRS complex, once ventricular pressure is high enough.
Atrial Repolarization (Incorrect)
Atrial repolarization occurs during the QRS complex itself but is masked by the large QRS wave on an ECG.
It does not occur immediately after the QRS complex.
Atrial Systole (Incorrect)
Atrial systole (contraction of the atria) occurs before the QRS complex, during the P wave .
The atria contract to fill the ventricles before ventricular contraction begins.
Ventricular Repolarization (Incorrect)
Ventricular repolarization is represented by the T wave , which occurs later , after the contraction and ejection phases.
The ductus arteriosus is a crucial fetal structure that shunts blood from the pulmonary artery to the aorta , bypassing the lungs. It eventually closes after birth to become the ligamentum arteriosum . Which embryonic aortic arch contributes to the development of this important vascular connection?
12 / 101
Category:
CVS – Embryology
Where does the ductus arteriosus arise from?
Why “Left Sixth Aortic Arch” is the Correct Answer?
The sixth aortic arches give rise to the pulmonary arteries and ductus arteriosus .
The left sixth aortic arch forms the ductus arteriosus , which connects the pulmonary trunk to the descending aorta .
After birth, the ductus arteriosus closes due to increased oxygen levels and reduced prostaglandins , forming the ligamentum arteriosum .
Why the Other Options Are Incorrect?
Left Fourth Aortic Arch – Incorrect
The left fourth aortic arch contributes to the aortic arch but not the ductus arteriosus .
Right Sixth Aortic Arch – Incorrect
The right sixth aortic arch forms part of the right pulmonary artery , but the ductus arteriosus arises from the left sixth aortic arch .
Left and Right Fourth Aortic Arches – Incorrect
The right fourth aortic arch forms the proximal part of the right subclavian artery , while the left fourth aortic arch forms part of the aortic arch .
Neither directly forms the ductus arteriosus .
Left and Right Sixth Aortic Arches – Incorrect
While both sixth aortic arches contribute to pulmonary arteries , only the left sixth aortic arch forms the ductus arteriosus .
“Why would it be dangerous for the heart to contract continuously without a relaxation phase? What physiological mechanism prevents this from happening?”
13 / 101
Category:
CVS – Physiology
Due to which of the following properties of cardiac muscle fibers, the cardiac contractile muscles do not exhibit tetanus?
Why Does Cardiac Muscle Not Exhibit Tetanus?
Tetanus occurs when a muscle is stimulated so rapidly that it does not relax between contractions , leading to a sustained contraction . Cardiac muscle cannot undergo tetanus because of its long absolute refractory period , which prevents rapid successive contractions.
Mechanism of the Long Absolute Refractory Period in Cardiac Muscle:
Prolonged Action Potential (Plateau Phase)
The cardiac action potential is prolonged (200–250 ms) due to the influx of calcium (Ca²⁺) through slow L-type calcium channels .
This creates a plateau phase that extends the duration of depolarization.
Long Absolute Refractory Period Prevents Summation
The absolute refractory period (ARP) is the time during which a new action potential cannot be initiated, no matter how strong the stimulus.
Because the refractory period lasts almost as long as the contraction itself , the heart must relax before another contraction can occur .
This prevents tetanus , ensuring the heart functions as a pump with coordinated contraction and relaxation.
Breakdown of Incorrect Options:
“They exclusively undergo aerobic respiration” → Incorrect
While cardiac muscle primarily relies on aerobic respiration , this is related to energy metabolism , not prevention of tetanus.
“They exclusively undergo anaerobic respiration” → Incorrect
Cardiac muscle does not rely exclusively on anaerobic respiration because continuous ATP generation is required for lifelong function.
Anaerobic respiration occurs only under pathological conditions like ischemia .
“They have well-developed sarcoplasmic reticulum” → Incorrect
While cardiac muscle has a sarcoplasmic reticulum (SR) for calcium storage, it is less developed than in skeletal muscle .
The long refractory period is the key factor preventing tetanus, not the SR structure.
“They are slow oxidative muscle fibers” → Incorrect
Cardiac muscle does share characteristics with slow oxidative fibers , but the reason it does not undergo tetanus is its long refractory period , not its fiber type.
Consider the role of cognitive biases in conflict resolution. Certain errors distort perception, making conflicts harder to resolve . Think of conflict resolution.
14 / 101
Category:
CVS – Community Medicine/ Behavioural Sciences
Conflict resolution is the process of ending a dispute and reaching an agreement that satisfies all parties involved. Since conflict is an essential part of being human, effective conflict resolution is not designed to avoid disagreements. Instead, conflict resolution skills are used to facilitate discussions, increase understanding, and control emotional responses. Which of the following does not pertain to the most cognitive errors that lead to unproductive and unresolved conflicts?
Conflict resolution involves recognizing and managing cognitive distortions that hinder productive discussions. The most common cognitive errors include:
Overconfidence → People often believe they are more correct than they actually are , leading to stubbornness and unwillingness to compromise.
Blaming → Assigning fault to others without self-reflection makes resolution difficult.
Self-serving fairness interpretation → People define fairness in a way that benefits them, leading to biased judgments and impasses.
Emotional volatility → Reacting emotionally rather than rationally escalates conflicts, making productive resolution difficult.
However, “emotional regulation” is not a cognitive error—it is a skill that helps resolve conflicts.
Why “Emotional Regulation” is the Correct Answer:
Emotional regulation refers to the ability to manage emotions effectively , stay calm , and respond thoughtfully in conflicts.
It helps prevent emotional volatility from escalating disputes.
Those with good emotional regulation are more likely to listen actively , consider alternative perspectives , and reach mutual agreements .
Why the Other Options Are Incorrect:
Emotional Volatility – Incorrect
Emotional instability can escalate conflicts and lead to impulsive reactions .
This is a major cognitive error because it clouds judgment and impairs communication .
Overconfidence – Incorrect
People often assume they are right and dismiss opposing viewpoints.
This is a classic cognitive bias that prevents open-minded discussion and compromise .
Blaming – Incorrect
Assigning responsibility to others without self-reflection leads to defensiveness.
This prevents parties from taking accountability and finding a resolution.
Self-Serving Fairness Interpretation – Incorrect
This occurs when individuals define “fairness” in a way that benefits them .
It is a cognitive distortion that prevents mutual understanding
For diagnosing myocardial infarction (MI), the earliest detected marker is not always the most specific . Since the question asks for a cardiac-specific marker at 4-5 hours post-MI , consider which biomarker is both early and highly specific to cardiac muscle .
15 / 101
Category:
CVS – BioChemistry
A patient comes to the emergency department 4-5 hours after a myocardial infarction. Which of the following cardiac markers will be detected at this point?
Why “Troponin I” is the Correct Answer?
Troponin I is highly specific to cardiac muscle and is not found in skeletal muscle, unlike myoglobin or CK-MB.
It begins to rise at 4-6 hours post-MI , peaks at 12-24 hours , and remains elevated for 7-10 days , making it the gold standard for MI diagnosis .
Cardiac troponins (I and T) are the preferred biomarkers in MI due to their high specificity and prolonged elevation .
Why the Other Options Are Incorrect?
Creatine Kinase-MB (CK-MB) – Incorrect
CK-MB is also cardiac-specific but rises slightly later (4-6 hours) and peaks at 12-24 hours .
It is less preferred because it can also be found in skeletal muscle injuries .
Pyruvate Kinase – Incorrect
Pyruvate kinase is an enzyme involved in glycolysis , not myocardial injury.
It has no diagnostic role in MI .
Lactic Dehydrogenase (LDH) – Incorrect
LDH rises much later (24-48 hours) post-MI, making it not useful for early detection .
It lacks specificity as it is found in multiple tissues .
Myoglobin – Incorrect (But Earliest Marker)
Myoglobin is the first biomarker to rise (1-4 hours post-MI) , but it is not cardiac-specific since it is also present in skeletal muscle.
It has low specificity for MI, making it less useful in definitive diagnosis .
“The heart must pause momentarily to allow the atria to complete their job before the ventricles take over. Where in the conduction system does this deliberate delay occur?”
16 / 101
Category:
CVS – Anatomy
Which part of the conduction pathway has the slowest conduction rate?
Understanding the Cardiac Conduction System and Conduction Velocity
The cardiac conduction system consists of specialized pathways that ensure the sequential contraction of the atria and ventricles. Each part of this system has a different conduction velocity , which is crucial for the timing and coordination of heartbeats.
Relative Conduction Speeds (Fastest to Slowest)
Conduction Pathway
Approximate Conduction Velocity
Purkinje fibers
2–4 m/s (fastest)
Bundle branches (RBB, LBB)
1.5–3 m/s
Atrial muscle & internodal pathways
1–1.2 m/s
Ventricular muscle
0.3–1 m/s
SA node
0.05 m/s
AV node
0.02–0.05 m/s (slowest)
Why is the AV Node the Slowest?
Physiological Purpose:
The delay at the AV node allows time for the atria to contract completely before ventricular contraction begins.
This ensures optimal ventricular filling before ejection occurs.
Cellular Structure:
The AV node has fewer gap junctions compared to other conduction tissues, reducing electrical coupling and slowing conduction.
Autonomic Regulation:
The AV node is heavily influenced by the autonomic nervous system :
Parasympathetic (vagal) stimulation → further slows conduction (negative dromotropic effect).
Sympathetic stimulation → increases conduction velocity (positive dromotropic effect).
Breakdown of Incorrect Options:
Purkinje Fibers → Incorrect
Purkinje fibers have the fastest conduction velocity (2–4 m/s) to ensure rapid and synchronized ventricular contraction.
Right Bundle Branch (RBB) → Incorrect
The bundle branches conduct impulses rapidly (1.5–3 m/s) , allowing coordinated activation of both ventricles.
SA Node → Incorrect
The SA node has a slow conduction velocity (0.05 m/s) but is still faster than the AV node .
It serves as the pacemaker but does not significantly delay conduction.
Internodal Pathway → Incorrect
The internodal pathways transmit impulses from the SA node to the AV node at a moderate speed (1–1.2 m/s) , which is faster than the AV node.
Atheroma plaques develop due to lipid accumulation in the arterial walls. Which lipid is most strongly associated with atherosclerosis , contributing to foam cell formation and plaque progression ?
17 / 101
Category:
CVS – BioChemistry
What is the most common type of lipid present in atheroma plaque?
An atheroma plaque is a hallmark of atherosclerosis , a condition characterized by the accumulation of lipids, inflammatory cells, and fibrous tissue in the arterial wall. The main lipid component of atherosclerotic plaques is cholesterol, particularly in the form of low-density lipoprotein (LDL).
Why “Low-Density Lipoprotein (LDL)” is the Correct Answer:
LDL is the primary carrier of cholesterol in the bloodstream and is responsible for delivering cholesterol to peripheral tissues.
In atherosclerosis, LDL infiltrates the arterial intima , where it undergoes oxidation , triggering an inflammatory response.
Macrophages engulf oxidized LDL, forming foam cells , which contribute to plaque development.
Elevated LDL cholesterol levels are directly linked to increased atherogenesis and cardiovascular disease risk.
Why the Other Options Are Incorrect:
Intermediate-Density Lipoprotein (IDL) – Incorrect
IDL is an intermediate form in the metabolism of very low-density lipoproteins (VLDL) to LDL .
While it can contribute to lipid transport, it is not the predominant lipid found in atheromatous plaques.
Apo-lipoprotein – Incorrect
Apolipoproteins are protein components of lipoproteins (such as ApoB-100 in LDL and ApoA in HDL).
They play a role in lipid transport but are not the primary lipid deposited in plaques .
High-Density Lipoprotein (HDL) – Incorrect
HDL is considered “good cholesterol” because it removes excess cholesterol from arterial walls and transports it to the liver for excretion.
Higher HDL levels are associated with a lower risk of atherosclerosis .
Cholesterol – Incorrect (but partially correct)
While cholesterol is a major component of atherosclerotic plaques , it does not circulate freely in the bloodstream .
LDL is the main carrier of cholesterol in the blood and is responsible for its deposition in plaques , making LDL the best answer .
“Which nerve/s work together in controlling both the diaphragm’s movement and the sensory reflex arc that triggers hiccups?”
18 / 101
Category:
CVS – Anatomy
Persistent hiccups can be caused by the irritation of which of the following nerves?
Understanding Hiccups and Their Neural Control
Hiccups (singultus ) are involuntary spasmodic contractions of the diaphragm , followed by a sudden closure of the glottis , producing the characteristic “hic” sound.
The two major nerves involved in hiccups are:
Phrenic Nerve (C3–C5):
Provides motor control to the diaphragm .
Irritation can cause diaphragmatic spasms , leading to hiccups.
Vagus Nerve (Cranial Nerve X):
Provides sensory input to the pharynx, larynx, esophagus, and diaphragm .
Irritation along its course can trigger the hiccup reflex arc .
Persistent hiccups (lasting more than 48 hours ) are often due to irritation of the vagus and/or phrenic nerve , which can result from:
Gastroesophageal reflux disease (GERD)
Neck or thoracic tumors
Pneumonia or pleuritis
Brainstem lesions affecting the vagus nerve
Breakdown of Incorrect Options:
Vagus Nerve → Partially Correct but Not the Best Answer
The vagus nerve alone can contribute to hiccups, but most cases involve both the vagus and phrenic nerves .
Choosing “Both vagus and phrenic nerves” is more accurate .
Glossopharyngeal Nerve → Incorrect
The glossopharyngeal nerve (CN IX) is mainly involved in swallowing and taste .
It does not control the diaphragm or play a direct role in hiccups.
Phrenic Nerve → Partially Correct but Not the Best Answer
While the phrenic nerve controls the diaphragm , hiccups are not just motor contractions but involve sensory reflex pathways through the vagus nerve.
Recurrent Laryngeal Nerve → Incorrect
The recurrent laryngeal nerve is a branch of the vagus nerve that innervates the larynx.
It may be involved in the glottis closure during hiccups but is not responsible for triggering the hiccup reflex .
“Which substance is released during allergic reactions and can cause sudden, life-threatening hypotension requiring immediate treatment?”
19 / 101
Category:
CVS – Pharmacology
Which of the following substances is most likely to cause a rapid drop in blood pressure?
Why Does Histamine Cause a Rapid Drop in Blood Pressure?
Histamine is a potent vasodilator that leads to a sudden drop in blood pressure by causing:
Vasodilation of Arterioles
Histamine binds to H1 receptors on vascular smooth muscle , causing relaxation and dilation of blood vessels .
This leads to a significant drop in systemic vascular resistance (SVR) , reducing blood pressure.
Increased Capillary Permeability
Histamine also increases capillary permeability , leading to fluid leakage from blood vessels into tissues .
This results in a decrease in effective circulating blood volume , further lowering blood pressure.
Role in Anaphylactic Shock
In severe allergic reactions (anaphylaxis ), massive histamine release from mast cells and basophils causes sudden hypotension , requiring emergency treatment with epinephrine .
Breakdown of Incorrect Options:
Angiotensin II → Incorrect
Angiotensin II is a potent vasoconstrictor that raises blood pressure , not lowers it.
It increases systemic vascular resistance (SVR) and promotes aldosterone secretion , leading to sodium and water retention , which raises blood pressure .
Thromboxane A2 → Incorrect
Thromboxane A2 (TXA2) is a vasoconstrictor and platelet aggregator .
It plays a role in clot formation and increasing blood pressure , not lowering it.
Norepinephrine → Incorrect
Norepinephrine (NE) is a strong vasoconstrictor that increases blood pressure by stimulating alpha-1 receptors on blood vessels.
It is used to treat hypotension, not cause it .
Nicotine → Incorrect
Nicotine stimulates the sympathetic nervous system (SNS) , causing vasoconstriction and increased blood pressure .
Chronic nicotine use leads to hypertension, not hypotension .
“Which ECG interval covers both the activation and recovery of the ventricles, from the moment they begin contracting until they fully relax?”
20 / 101
Category:
CVS – Physiology
Ventricular depolarization and repolarization is shown by which of the following?
Understanding the ECG and the Q-T Interval
An electrocardiogram (ECG/EKG) represents the electrical activity of the heart. Each wave and interval corresponds to different phases of cardiac depolarization and repolarization .
Breakdown of the Q-T Interval:
The Q-T interval spans from the beginning of the Q wave to the end of the T wave .
It represents both ventricular depolarization (QRS complex) and ventricular repolarization (T wave).
A prolonged Q-T interval can indicate arrhythmia risks , such as Torsades de Pointes .
Breakdown of Incorrect Options:
T Wave → Incorrect
The T wave represents only ventricular repolarization , not depolarization.
It marks the phase where the ventricles recover from contraction .
QRS Complex → Incorrect
The QRS complex represents ventricular depolarization only , not repolarization.
It corresponds to ventricular contraction .
P Wave → Incorrect
The P wave represents atrial depolarization , not ventricular activity.
It indicates atrial contraction before the ventricles are activated.
P-R Interval → Incorrect
The P-R interval represents the time taken for the electrical impulse to travel from the SA node through the atria and AV node to the ventricles .
It does not include ventricular repolarization.
Shock can be classified based on its underlying cause . Which type of shock is primarily associated with infection and systemic inflammation ?
21 / 101
Category:
CVS – Physiology
A young patient presented to the emergency department with signs of shock after a bacterial infection. Which of the following types of shock is the most likely to be found in this patient?
Why “Septic Shock” is the Correct Answer?
Why the Other Options Are Incorrect?
Cardiogenic Shock – Incorrect
Cardiogenic shock is due to heart failure (e.g., myocardial infarction, arrhythmias, or cardiac tamponade) .
This patient has an infection-related shock, not heart failure .
Neurogenic Shock – Incorrect
Neurogenic shock occurs due to spinal cord injury , causing loss of sympathetic tone and severe hypotension .
This patient has signs of infection, not spinal trauma .
Anaphylactic Shock – Incorrect
Anaphylactic shock is caused by a severe allergic reaction (e.g., peanuts, bee stings, medications) , leading to histamine release and vasodilation .
This patient’s shock is related to infection, not an allergic reaction .
Hypovolemic Shock – Incorrect
Hypovolemic shock results from fluid loss (e.g., hemorrhage, severe dehydration, burns) .
While sepsis can cause some fluid shifts , the primary mechanism here is infection-induced vasodilation , not volume depletion.
The tunica media of the aorta must withstand high pressure and pulsatile blood flow . Which structural component provides the necessary thing to allow the aorta to stretch during systole and recoil during diastole, maintaining continuous blood flow?
22 / 101
Category:
CVS – Histology
What is the most abundant component of tunica media of the aorta?
The aorta is a large elastic artery , and its tunica media contains layers of elastic fibers that allow it to stretch and recoil with each heartbeat.
Why “Elastin” is the Correct Answer:
Elastin is the most abundant component of the tunica media of the aorta.
It forms concentric, wavy elastic lamellae , which allow the aorta to expand during systole (when blood is ejected from the heart) and recoil during diastole , ensuring continuous blood flow.
This elasticity reduces the workload on the heart by dampening pressure fluctuations .
Elastic fibers predominate in large elastic arteries like the aorta but are less prominent in muscular arteries.
Why the Other Options Are Incorrect:
Cholesterol – Incorrect
Cholesterol is not a structural component of the aorta’s tunica media.
It is involved in atherosclerosis , where it accumulates in plaques but is not a major normal component of the tunica media.
Smooth Muscles – Incorrect (but present)
Smooth muscle cells are present in the tunica media , but their role is secondary to elastin in large elastic arteries.
In muscular arteries (e.g., radial artery, femoral artery) , smooth muscle is more dominant for vasoconstriction and regulation of blood flow.
In the aorta, elastin is more abundant than smooth muscle .
Collagen – Incorrect
Collagen provides tensile strength and is more abundant in the tunica adventitia , the outermost layer of the aorta.
In the tunica media, collagen is present in small amounts but does not provide elasticity .
Reticular Fibers – Incorrect
Reticular fibers (a type of collagen fiber ) are mainly found in lymphoid tissues and provide a supportive network .
They are not a major component of the tunica media .
“The atria and ventricles are separated by a fibrous barrier that blocks electrical conduction. What structure allows impulses to pass through this barrier?”
23 / 101
Category:
CVS – Physiology
Which of the following is the only electrical connection between the atria and the ventricles?
Understanding the Electrical Connection Between the Atria and Ventricles
The atria and ventricles are electrically insulated from each other by the fibrous skeleton of the heart .
The only pathway that allows electrical impulses to travel from the atria to the ventricles is the Bundle of His (atrioventricular bundle) , which originates from the AV node .
Pathway of Cardiac Conduction:
Sinoatrial (SA) Node → Generates impulse, spreading through atria.
Atrioventricular (AV) Node → Delays impulse to allow atrial contraction.
Bundle of His (AV Bundle) → The only electrical connection between atria and ventricles .
Right and Left Bundle Branches → Conduct impulse through the interventricular septum.
Purkinje Fibers → Spread impulse to ventricular myocardium for contraction.
Breakdown of Incorrect Options:
Atrioventricular Node → Incorrect
The AV node delays conduction , but it does not directly transmit the impulse to the ventricles .
It passes the impulse to the Bundle of His , which is the true electrical connection.
Atrioventricular Delay → Incorrect
The AV delay is a physiological phenomenon at the AV node to allow atrial contraction before ventricular contraction.
However, it is not a physical structure that connects the atria and ventricles .
Atrioventricular Valve → Incorrect
The AV valves (tricuspid and mitral) prevent backflow of blood but do not conduct electrical impulses .
Bundle Branches → Incorrect
The bundle branches conduct impulses to the ventricles, but they are not the first structure connecting the atria and ventricles .
They originate from the Bundle of His , which is the actual electrical connection.
“Which ion movement would make the SA node more negative and harder to excite, leading to a slower heart rate?”
24 / 101
Category:
CVS – Physiology
Vagal stimulation results in the slowing of the heart rate. This is due to the increased permeability of the sinoatrial nodal fiber membrane to which of the following?
How Vagal Stimulation Slows Heart Rate (Negative Chronotropic Effect)
Vagal stimulation (parasympathetic activation via the vagus nerve ) slows the heart rate by increasing potassium (K⁺) permeability in the sinoatrial (SA) node .
Mechanism:
Acetylcholine (ACh) Release
The vagus nerve releases acetylcholine (ACh) , which binds to muscarinic (M2) receptors in the SA node .
Increased Potassium Conductance
ACh opens potassium channels (IK-ACh) , allowing potassium efflux (K⁺ leaving the cell).
This hyperpolarizes the SA node , making it less excitable and slowing down spontaneous depolarization.
Decreased Pacemaker Activity
Hyperpolarization increases the time needed for the SA node to reach threshold , thereby slowing heart rate (negative chronotropic effect).
Breakdown of Incorrect Options:
Sodium and Calcium → Incorrect
Sodium (Na⁺) and calcium (Ca²⁺) influx are responsible for pacemaker depolarization in the SA node.
Vagal stimulation inhibits these currents, but the main effect is through increased potassium efflux .
Calcium → Incorrect
Calcium (Ca²⁺) plays a major role in depolarization of the SA node , but vagal stimulation does not increase Ca²⁺ permeability .
Instead, it reduces Ca²⁺ influx , contributing to a slower depolarization rate.
Chloride → Incorrect
Chloride (Cl⁻) does not play a significant role in pacemaker activity.
Its movement does not contribute to vagally-induced bradycardia.
Sodium → Incorrect
Sodium (Na⁺) is essential for pacemaker activity , but vagal stimulation does not increase Na⁺ permeability .
Instead, it reduces Na⁺ and Ca²⁺ influx , slowing the rate of depolarization.
“Some congenital conditions affect not just the heart but also other organ systems. Think about how structural development in early embryogenesis might influence overall anatomical positioning.”
25 / 101
Category:
CVS – Pathology
Dextrocardia is commonly associated with which of the following conditions?
Understanding Dextrocardia and Its Association with Situs Inversus
Dextrocardia refers to a congenital condition where the heart is positioned in the right side of the chest , rather than the left.
It often occurs as part of situs inversus , a condition in which all major visceral organs are mirrored from their normal anatomical positions.
Types of Dextrocardia:
Dextrocardia with Situs Inversus (Situs Inversus Totalis):
The heart and abdominal organs are completely mirrored (right-sided heart, liver on the left, stomach on the right).
Usually, there are no functional cardiovascular abnormalities , and individuals may be asymptomatic.
Associated with Kartagener’s syndrome (Primary Ciliary Dyskinesia) , which includes chronic sinusitis, bronchiectasis, and infertility due to ciliary dysfunction.
Dextrocardia with Situs Solitus:
The heart is located on the right side, but the abdominal organs remain in their normal positions.
More likely associated with congenital heart defects , including Tetralogy of Fallot, transposition of the great arteries, and ventricular septal defects (VSDs).
Breakdown of Incorrect Options:
Tetralogy of Fallot → Incorrect
Tetralogy of Fallot (TOF) is a congenital heart defect with pulmonary stenosis, right ventricular hypertrophy, overriding aorta, and VSD .
While TOF can occur with dextrocardia, it is not the most common association —it occurs more frequently with situs solitus with dextrocardia rather than situs inversus .
Aortic Stenosis → Incorrect
Aortic stenosis (narrowing of the aortic valve) is unrelated to dextrocardia.
It usually occurs due to bicuspid aortic valve or age-related calcification , rather than abnormal cardiac positioning.
Coarctation of Aorta → Incorrect
Coarctation of the aorta (CoA) is a congenital narrowing of the aortic arch, commonly associated with Turner syndrome .
It does not have a strong link to dextrocardia or situs inversus .
Patent Ductus Arteriosus (PDA) → Incorrect
PDA is an abnormal persistence of the ductus arteriosus , leading to left-to-right shunting of blood.
While PDA can be associated with congenital heart defects, it is not a primary feature of dextrocardia or situs inversus .
“This macronutrient requires bile salts for proper digestion and absorption. Without bile storage, its incomplete absorption leads to greasy stools.”
26 / 101
Category:
CVS – BioChemistry
A middle-aged patient who underwent cholecystectomy presented to the outpatient department with steatorrhea. Improper absorption of which of the following could be the reason for this condition?
Steatorrhea refers to the presence of excess fat in the stool , which makes the stool pale, bulky, greasy, and difficult to flush . The condition arises due to impaired digestion or absorption of lipids (fats) , commonly associated with bile deficiency following a cholecystectomy (gallbladder removal).
Role of the Gallbladder in Lipid Digestion:
The gallbladder stores and concentrates bile , which is produced by the liver.
During meals, particularly fatty meals , the gallbladder releases bile into the small intestine .
Bile salts emulsify fats , breaking them down into smaller droplets to aid in lipase enzyme activity for digestion and absorption.
After cholecystectomy , bile is continuously released in a more diluted and unregulated manner , reducing its effectiveness in fat digestion.
This results in fat malabsorption , leading to steatorrhea .
Why the Other Options Are Incorrect:
Carbohydrates (Incorrect)
Carbohydrate digestion is mainly dependent on amylase (salivary & pancreatic) and brush border enzymes (e.g., lactase, maltase) .
A gallbladder removal does not significantly affect carbohydrate digestion.
Proteins (Incorrect)
Protein digestion primarily depends on gastric pepsin and pancreatic proteases (e.g., trypsin, chymotrypsin, peptidases) .
Bile is not essential for protein digestion, so cholecystectomy does not lead to protein malabsorption .
Vitamins (Incorrect, but Partially Relevant)
Fat-soluble vitamins (A, D, E, K) require bile for absorption, so their deficiency can occur secondary to lipid malabsorption .
However, steatorrhea itself is directly due to fat malabsorption , not vitamin deficiency.
Iron (Incorrect)
Iron absorption occurs mainly in the duodenum and does not require bile salts.
Iron deficiency is not a cause of steatorrhea.
Clinical Correlation:
Post-cholecystectomy syndrome can cause bile acid diarrhea and fat malabsorption .
Pancreatic insufficiency (e.g., chronic pancreatitis) can also cause steatorrhea due to lack of lipase .
Patients with steatorrhea may experience weight loss and fat-soluble vitamin deficiencies .
“Which lipoprotein is the largest, least dense, and primarily responsible for transporting dietary fats absorbed from the intestines?”
27 / 101
Category:
CVS – BioChemistry
Which of the following lipoproteins has the least percentage of protein?
Understanding Lipoprotein Composition
Lipoproteins are complexes of lipids and proteins that transport fats in the bloodstream. The proportion of protein and lipid content varies among different lipoproteins.
Key Lipoprotein Characteristics:
Chylomicrons have the least percentage of protein and the highest percentage of triglycerides.
The protein content of a lipoprotein determines its density —higher protein = higher density.
Lipoprotein Composition Table (Approximate Values):
Lipoprotein
Protein Content (%)
Lipid Content (%)
Density
Chylomicrons
1-2%
98-99%
Lowest (Least Dense)
Very Low-Density Lipoprotein (VLDL)
10-15%
85-90%
Low
Intermediate-Density Lipoprotein (IDL)
20-30%
70-80%
Medium
Low-Density Lipoprotein (LDL)
25-30%
70-75%
High
High-Density Lipoprotein (HDL)
50%
50%
Highest (Most Dense)
Why is Chylomicron the Correct Answer?
Chylomicrons contain the least percentage of protein (~1-2%) and the highest percentage of triglycerides (~85-90%) .
They are the largest and least dense lipoproteins , primarily involved in dietary fat transport from the intestines to tissues.
Breakdown of Incorrect Options:
Very Low-Density Lipoprotein (VLDL) → Incorrect
VLDL contains more protein (~10-15%) than chylomicrons.
Intermediate-Density Lipoprotein (IDL) → Incorrect
IDL has higher protein content (~20-30%) than chylomicrons.
Low-Density Lipoprotein (LDL) → Incorrect
LDL contains even more protein (~25-30%) , as it transports cholesterol to tissues.
High-Density Lipoprotein (HDL) → Incorrect
HDL has the highest protein content (~50%) and is the most dense lipoprotein .
“The superior vena cava collects blood from the upper body and is formed by the joining of two major veins. Where on what side of the thorax does this venous merging occur?”
28 / 101
Category:
CVS – Embryology
Which of the following is the point of formation of the superior vena cava?
Understanding the Formation of the Superior Vena Cava (SVC)
The superior vena cava (SVC) is a large vein that returns deoxygenated blood from the upper body (head, neck, upper limbs, and thorax) to the right atrium of the heart.
Formation of the SVC:
The SVC is formed by the union of the right and left brachiocephalic veins.
This occurs at the level of the right first costal cartilage .
The SVC then descends vertically behind the first and second right costal cartilages before entering the right atrium at the level of the third costal cartilage .
Breakdown of Incorrect Options:
Left Third Costal Cartilage → Incorrect
The left side does not contribute directly to the final SVC formation.
The SVC is primarily a right-sided structure , and the left brachiocephalic vein drains into it.
Left First Costal Cartilage → Incorrect
The left brachiocephalic vein crosses behind the manubrium to join the right brachiocephalic vein, but the formation of the SVC itself happens on the right side .
Left Second Costal Cartilage → Incorrect
Again, the left brachiocephalic vein is involved , but the SVC forms at the right first costal cartilage .
Right Third Costal Cartilage → Incorrect
The SVC enters the right atrium at the level of the third costal cartilage , but its formation occurs higher up, at the right first costal cartilage .
Think about how the body responds to transient ischemia and how mast cells in the myocardium might contribute to pain sensation. What mediator is commonly associated with both allergic reactions and vasodilation, yet can also be involved in localized ischemic responses?
29 / 101
Category:
CVS – Pathology
A 48-year-old man experienced deep localized chest pain associated with dyspnea and sweating which was relieved with rest. According to him, the pain developed after a long tiring day at his job. Pain in this condition is induced due to the release of which of the following mediators?
The patient in this scenario has exertional angina , a condition in which myocardial oxygen demand temporarily exceeds supply due to underlying coronary artery disease. The pain is triggered by physical exertion and relieved by rest , suggesting a transient ischemic episode.
While adenosine is a well-known mediator of ischemic pain , histamine also plays a role in coronary vasodilation and nociceptive signaling in ischemic conditions.
Why “Histamine” is the Correct Answer:
Histamine is released from mast cells in response to transient ischemia.
It acts on H1 and H2 receptors in the coronary vasculature, leading to vasodilation , which is an attempt to improve oxygen delivery.
Histamine can stimulate sensory nerve endings , contributing to chest pain perception in angina .
Research suggests that histamine may sensitize cardiac nociceptors , making the heart more sensitive to ischemic pain.
Why the Other Options Are Incorrect:
Epinephrine – Incorrect
Epinephrine is a sympathetic neurotransmitter that increases heart rate and myocardial oxygen demand .
While it can contribute to angina indirectly by increasing cardiac workload, it does not directly mediate ischemic pain .
Interleukins – Incorrect
Interleukins (such as IL-1, IL-6) are inflammatory cytokines primarily involved in chronic inflammation and immune responses.
They play a role in atherosclerosis progression but are not immediate mediators of ischemic pain .
Adenosine – Incorrect (Commonly Thought to Be Correct, but Not in This Case)
Adenosine is a well-known ischemic pain mediator , but in this question, histamine is emphasized .
Although adenosine activates pain receptors , it also induces coronary vasodilation , helping relieve ischemia rather than directly triggering pain in this context.
Histamine’s role in nociceptive sensitization in cardiac tissue is the key distinction here.
Serotonin – Incorrect
Serotonin (5-HT) is a vasoconstrictor involved in platelet aggregation and coronary vasospasm .
While it plays a role in unstable angina and variant angina , it is not the primary mediator of stable exertional angina pain .
“Inferior wall infarctions are seen on ECG leads.. Which artery is primarily responsible for supplying this region in the majority of people?”
30 / 101
Category:
CVS – Anatomy
Which of the following arteries is responsible for the myocardial infarction of the inferior wall of the myocardium?
Understanding Inferior Myocardial Infarction and Coronary Supply
An inferior wall myocardial infarction (MI) occurs due to ischemia affecting the inferior (diaphragmatic) surface of the heart , which is primarily supplied by the right coronary artery (RCA) in most individuals .
Coronary Supply to the Inferior Wall of the Heart:
The inferior wall of the myocardium corresponds mainly to the inferior portion of the left ventricle and part of the right ventricle .
The right coronary artery (RCA) supplies the inferior wall via its terminal branches, particularly the posterior interventricular artery (posterior descending artery, PDA) in right-dominant circulation (which is present in ~85% of people).
Occlusion of the RCA leads to inferior wall infarction , often seen with ST-segment elevation in leads II, III, and aVF on ECG .
Breakdown of Incorrect Options:
Anterior Interventricular Artery (Left Anterior Descending, LAD) → Incorrect
The LAD supplies the anterior wall , the anterior interventricular septum , and the apex of the heart.
Occlusion of the LAD leads to an anterior wall MI, not an inferior wall MI .
ECG changes would be seen in V1–V4 , not in the inferior leads.
Left Coronary Artery (LCA) → Incorrect
The LCA branches into the LAD and circumflex arteries , but it does not directly supply the inferior wall in most people.
While the circumflex artery may supply the inferior wall in left-dominant circulation (~15% of cases), the RCA is the more common culprit.
Posterior Interventricular Artery (PDA) → Incorrect
The PDA supplies the posterior interventricular septum and part of the inferior wall , but it usually branches from the RCA in right-dominant circulation .
Since the RCA is the parent artery , the correct answer remains the right coronary artery .
Right Marginal Artery → Incorrect
The right marginal artery is a branch of the RCA and mainly supplies the right ventricular free wall , not the inferior wall of the left ventricle.
It is not the main artery responsible for inferior wall MI .
n chronically bedridden patients , what would be the most severe and widespread manifestation.
31 / 101
Category:
CVS – Pathology
One of the presentations in right-sided heart failure patients, who are chronically bedridden, is anasarca. What does the term anasarca mean?
Why “Generalized Edema” is the Correct Answer?
In right-sided heart failure , blood backs up into the systemic venous circulation , leading to:
Increased central venous pressure
Capillary hydrostatic pressure rise → Fluid leaks into tissues
Reduced renal perfusion → Activation of the renin-angiotensin-aldosterone system (RAAS) → Sodium and water retention
Bedridden patients develop anasarca because gravity distributes excess fluid evenly , leading to widespread swelling .
Why the Other Options Are Incorrect?
Ascites – Incorrect
Ascites is the accumulation of fluid in the peritoneal cavity , but it does not describe generalized edema .
Right-sided heart failure can cause ascites , but anasarca is more severe and widespread .
Renal Congestion – Incorrect
Right-sided heart failure leads to renal congestion due to venous stasis , but it is not the definition of anasarca .
Chronic congestion can contribute to sodium and water retention , worsening anasarca , but renal congestion alone is not the primary term.
Left Ventricular Hypertrophy – Incorrect
Left ventricular hypertrophy (LVH) is a compensatory response to increased afterload (e.g., hypertension or aortic stenosis) .
LVH is not related to right-sided heart failure or anasarca .
Pulmonary Hypertension – Incorrect
Pulmonary hypertension leads to right-sided heart failure (cor pulmonale) but does not directly define anasarca .
Anasarca results from systemic venous congestion , while pulmonary hypertension is an upstream cause.
an important fetal cardiac structure that allows oxygenated blood to bypass the lungs by shunting from the right atrium to the left atrium .
32 / 101
Category:
CVS – Embryology
Which of the following gives the correct location of the foramen ovale?
Why “Interatrial Septum” is the Correct Answer?
The foramen ovale is a normal fetal opening in the interatrial septum , allowing blood to flow from the right atrium to the left atrium , bypassing the non-functional fetal lungs.
After birth, when the newborn starts breathing, left atrial pressure increases, forcing the foramen ovale to close .
It eventually seals permanently , forming the fossa ovalis , a depression in the interatrial septum.
Why the Other Options Are Incorrect?
Coronary Sulcus – Incorrect
The coronary sulcus (atrioventricular groove) is an external groove that separates the atria from the ventricles and houses the coronary arteries and veins .
It has no connection to the foramen ovale .
Anterior Interventricular Septum – Incorrect
The interventricular septum separates the right and left ventricles , not the atria.
The foramen ovale is located in the interatrial septum , not the ventricular septum.
Posterior Interventricular Septum – Incorrect
Similar to the anterior interventricular septum, the posterior interventricular septum is part of the division between the ventricles , not the atria.
The foramen ovale has no anatomical relation to the ventricular septum .
Coronary Sinus – Incorrect
The coronary sinus is the largest vein draining deoxygenated blood from the heart muscle into the right atrium .
It is located in the posterior part of the coronary sulcus , not the interatrial septum.
“Some blood vessels are too thick for diffusion alone to provide nourishment.”
33 / 101
Category:
CVS – Embryology
Vasa vasorum are predominantly found in the large arteries and veins. Which of the following best describes vasa vasorum?
Understanding Vasa Vasorum
Vasa vasorum (Latin for “vessels of the vessels”) are tiny blood vessels that supply the walls of large arteries and veins .
They provide oxygen and nutrients to the outer layers of large blood vessels (such as the aorta, vena cava, and pulmonary arteries) because diffusion alone is insufficient to nourish thick vessel walls .
Where Are Vasa Vasorum Found?
In Large Arteries:
Present in the tunica adventitia (outermost layer) and extend into the tunica media .
They are especially prominent in elastic arteries (e.g., aorta) and large muscular arteries .
In Large Veins:
More extensive than in arteries due to lower oxygen content in venous blood , requiring additional nutrient support.
Found in large veins like the inferior vena cava (IVC) and superior vena cava (SVC) .
Breakdown of Incorrect Options:
Blood Vessels of the Myocardium → Incorrect
The coronary arteries , not the vasa vasorum, supply blood to the myocardium.
The vasa vasorum is specifically associated with vessel walls, not heart muscle .
Nerves That Supply the Blood Vessels → Incorrect
Nervi vasorum are the nerves that innervate blood vessels and help regulate vasoconstriction and vasodilation.
Vasa vasorum are blood vessels, not nerves .
Blood Vessels of the Endocardium → Incorrect
The endocardium (inner lining of the heart) is primarily supplied by diffusion from the heart chambers , not by vasa vasorum.
The coronary arteries supply deeper heart structures , but they are not vasa vasorum.
Nerves of the Endocardium → Incorrect
The endocardium contains autonomic nerve fibers , but these are not vasa vasorum.
The correct term for vessel-related nerves is nervi vasorum , not vasa vasorum.
“Which hormone promotes lipid storage and cholesterol synthesis, playing a key role in metabolic syndrome and hyperlipidemia?”
34 / 101
Category:
CVS – BioChemistry
Which of the following hormones increases the endogenous synthesis of cholesterol?
How Insulin Increases Endogenous Cholesterol Synthesis
Insulin promotes lipogenesis , including cholesterol synthesis , by upregulating HMG-CoA reductase , the rate-limiting enzyme in cholesterol biosynthesis .
This occurs in the liver , where insulin stimulates the conversion of acetyl-CoA into cholesterol .
Increased insulin levels (e.g., in obesity or type 2 diabetes) are associated with increased cholesterol synthesis , contributing to hyperlipidemia .
Breakdown of Incorrect Options:
Estrogen → Incorrect
Estrogen does not directly increase cholesterol synthesis .
Instead, estrogen enhances HDL levels and lowers LDL , which is beneficial for lipid metabolism.
Glucagon → Incorrect
Glucagon inhibits HMG-CoA reductase , thereby reducing cholesterol synthesis .
It opposes insulin’s effects and promotes lipid breakdown (lipolysis ).
Glucocorticoids → Incorrect
Glucocorticoids (e.g., cortisol ) promote lipolysis and gluconeogenesis , but they do not directly increase cholesterol synthesis .
However, chronic glucocorticoid use may lead to dyslipidemia and increased LDL levels .
Progesterone → Incorrect
Progesterone is a precursor for steroid hormones , but it does not regulate cholesterol synthesis directly .
It is synthesized from cholesterol , but it does not increase its production .
“During fetal development, certain perforations ensure continuous blood flow between the atria before complete septation. What structure emerges from the merging of these perforations?”
35 / 101
Category:
CVS – Embryology
Perforations in septum primum coalesce before the closure of foramen primum. This leads to the formation of which of the following?
Understanding Atrial Septation and the Role of Foramen Secundum
During embryonic heart development, the atrial septum forms to separate the right and left atria. This process involves two major septa:
Septum Primum:
Grows downward from the roof of the primitive atrium .
Leaves an initial opening called the foramen primum , which allows blood to shunt from the right to the left atrium.
Before the foramen primum completely closes, perforations appear in the septum primum , which later coalesce to form the foramen secundum .
Foramen Secundum:
It ensures continued right-to-left blood flow , allowing fetal circulation to bypass the non-functional lungs.
It persists until birth when the pressure changes in the heart lead to closure of the foramen ovale.
Breakdown of Incorrect Options:
Septum Secundum → Incorrect
The septum secundum develops later, to the right of the septum primum, and partially covers the foramen secundum.
It helps form the foramen ovale but is not directly formed by perforations in the septum primum .
Sinus Venosus → Incorrect
The sinus venosus is an embryonic structure that contributes to the right atrium, superior vena cava, and coronary sinus , but it is not related to the formation of the foramen secundum .
Coronary Sinus → Incorrect
The coronary sinus forms from the left horn of the sinus venosus , not from the septum primum.
It is involved in venous drainage of the heart , not atrial septation.
Foramen Ovale → Incorrect
The foramen ovale forms when the septum secundum overlaps the foramen secundum, allowing fetal blood shunting.
The foramen secundum precedes the foramen ovale , making this answer incorrect.
“Think of an electrical triangle where each side represents a lead. The sum of two sides must always give the third. This fundamental ECG law helps ensure consistency in readings.”
36 / 101
Category:
CVS – Physiology
A medical student is given an ECG in which lead I and III potentials are 0.5 mv and 0.7 mv, respectively. Taking these values into account, what is the potential of lead II?
This question tests your understanding of Einthoven’s Law , which states:
Lead I+Lead III=Lead II\text{Lead I} + \text{Lead III} = \text{Lead II}Lead I+Lead III=Lead II
This fundamental principle in electrocardiography (ECG) is derived from the standard limb lead system.
Step-by-Step Calculation:
We are given:
Lead I=0.5 mV\text{Lead I} = 0.5 \text{ mV}Lead I=0.5 mV Lead III=0.7 mV\text{Lead III} = 0.7 \text{ mV}Lead III=0.7 mV
Using Einthoven’s Law :
Lead II=Lead I+Lead III\text{Lead II} = \text{Lead I} + \text{Lead III}Lead II=Lead I+Lead III Lead II=0.5+0.7\text{Lead II} = 0.5 + 0.7Lead II=0.5+0.7 Lead II=1.2 mV\text{Lead II} = 1.2 \text{ mV}Lead II=1.2 mV
Thus, the correct answer is 1.2 mV .
Why the Correct Answer is Right:
Einthoven’s triangle states that the three standard limb leads are mathematically related. By summing Lead I and Lead III, we directly obtain the value of Lead II.
This is a basic application of vector addition in electrocardiography, ensuring consistency across the ECG leads.
Why the Other Options are Incorrect:
0 mV (Incorrect)
This would imply that Lead I and Lead III are equal and opposite in magnitude, which contradicts the given values.
-0.2 mV (Incorrect)
A negative value would suggest that Lead III is smaller than Lead I in a way that their sum does not reach 1.2 mV. However, both leads are positive in this case.
-1.2 mV (Incorrect)
A completely opposite sign would contradict the principle of Einthoven’s Law, as we are adding two positive values.
0.2 mV (Incorrect)
This is much lower than the expected value calculated using Einthoven’s Law, and there is no basis for subtracting the given values instead of adding them.
“Imagine squeezing a garden hose and then letting go—what happens to the water flow immediately after the pressure is released?”
37 / 101
Category:
CVS – Physiology
Normally, when the pressure blocking the blood vessels is released, there is a greater blood flow to the tissues. What is this phenomenon called?
Reactive hyperemia is the increase in blood flow to a tissue following a period of ischemia (temporary blockage of blood supply) . When the obstruction in the blood vessel is removed , there is a sudden surge in blood flow due to the accumulation of vasodilatory metabolites (e.g., CO₂, lactate, and adenosine) and the restoration of oxygen supply .
Mechanism of Reactive Hyperemia:
Blood flow is temporarily blocked due to an obstruction (e.g., a tourniquet, vascular occlusion, or arterial constriction).
Oxygen levels drop , and metabolic waste products accumulate in the affected tissue.
Local vasodilation occurs due to the release of vasoactive substances like adenosine, nitric oxide, and CO₂ .
When the blockage is removed , blood rushes back into the tissue at a higher rate than normal to restore oxygen levels and clear metabolic waste .
After a short period , blood flow returns to normal once homeostasis is reestablished.
Why the Other Options Are Incorrect:
Hypovolemic Shock (Incorrect)
Hypovolemic shock occurs due to severe fluid loss (e.g., hemorrhage, dehydration) leading to low blood volume and inadequate tissue perfusion .
It is not related to temporary occlusion and subsequent blood flow increase .
Angiogenesis (Incorrect)
Angiogenesis is the formation of new blood vessels , typically in response to chronic hypoxia (e.g., tumor growth, chronic ischemia, wound healing).
It does not occur immediately after a temporary vascular occlusion.
Active Hyperemia (Incorrect)
Active hyperemia occurs when tissue metabolic activity increases , leading to a rise in blood flow to meet demand (e.g., during exercise or digestion).
Reactive hyperemia is different because it is a response to a temporary blood flow blockage , not an increase in metabolic activity.
Tissue Vascularity (Incorrect)
Tissue vascularity refers to the density of blood vessels in a given tissue.
It is not a transient response to blood flow occlusion .
Clinical Examples of Reactive Hyperemia:
Removing a tourniquet : Blood rushes back into the limb after release.
Raynaud’s phenomenon : Following vasospasm, there is increased blood flow, causing redness and warmth .
Post-ischemic reperfusion injury : Sudden reperfusion can cause oxidative stress and inflammation in previously ischemic tissue.
Think about the difference between actions taken before a crisis to reduce its impact and actions taken after a crisis to address its consequences. Which of the options aligns with the idea of planning and preparation to minimize harm before a disaster occurs?”
38 / 101
Category:
CVS – Community Medicine/ Behavioural Sciences
Japan is a part of the Pacific Ring of Fire, the most active earthquake belt in the world, and accounts for 20% of earthquakes worldwide. Because of this, the government of Japan has designed buildings in a way that reduces loss as a result of an earthquake. Advanced precautionary measures have reduced the death toll and damage as a result of an earthquake. Which of these best explains the timely precautionary measures made by the government of Japan?
Why Crisis Intervention is Correct:
Crisis intervention refers to the proactive measures taken to prevent or mitigate the effects of a crisis before it occurs. In the context of Japan, the government has implemented advanced precautionary measures, such as earthquake-resistant building designs, early warning systems, and public education programs, to reduce the impact of earthquakes. These actions are aimed at minimizing damage, saving lives, and ensuring public safety. Crisis intervention is a systematic approach to managing potential disasters, and Japan’s efforts align perfectly with this concept.
Why the Other Options Are Incorrect:
Coping Skills:
Coping skills refer to the psychological strategies individuals use to manage stress or adversity after a traumatic event has occurred. While coping skills are important for individuals dealing with the aftermath of an earthquake, they do not describe the government’s preemptive actions to reduce earthquake damage. Coping skills are reactive, not proactive.
Post-Traumatic Growth:
Post-traumatic growth refers to the positive psychological changes that individuals may experience after overcoming a traumatic event. This concept is about personal development following a crisis, not about the government’s efforts to prevent or mitigate the effects of a disaster. It is unrelated to precautionary measures.
Post-Traumatic Stress Disorder (PTSD):
PTSD is a mental health condition that can develop after someone experiences or witnesses a traumatic event. It is a psychological consequence of a crisis, not a strategy or measure taken to prevent or reduce the impact of a crisis. This option is irrelevant to the government’s precautionary actions.
Resolution of Crisis:
Resolution of crisis refers to the steps taken to address and resolve a crisis after it has occurred. While this is an important part of disaster management, it does not describe the preemptive measures taken by the Japanese government to reduce earthquake damage. Resolution of crisis is a reactive process, not a proactive one.
“Which enzyme is the key regulatory checkpoint in cholesterol synthesis and the primary target of cholesterol-lowering medications?”
39 / 101
Category:
CVS – BioChemistry
Which of the following is the rate-limiting enzyme in cholesterol synthesis?
Understanding Cholesterol Synthesis and the Rate-Limiting Step
Cholesterol synthesis is a multi-step biochemical pathway that takes place primarily in the liver . The rate-limiting step in this process is catalyzed by 3-hydroxy-3-methylglutaryl-coenzyme A (HMG-CoA) reductase .
Why is HMG-CoA Reductase the Rate-Limiting Enzyme?
Catalytic Function:
HMG-CoA reductase converts HMG-CoA into mevalonate , which is a critical early step in cholesterol biosynthesis.
This step commits acetyl-CoA to cholesterol synthesis .
Regulation of Cholesterol Synthesis:
Feedback Inhibition: High levels of cholesterol inhibit HMG-CoA reductase activity to prevent excess cholesterol synthesis.
Hormonal Control:
Insulin stimulates HMG-CoA reductase → increases cholesterol synthesis .
Glucagon inhibits it → decreases cholesterol synthesis .
Statin Drugs:
Statins (e.g., atorvastatin, simvastatin) inhibit HMG-CoA reductase , reducing cholesterol synthesis to lower LDL cholesterol levels.
Breakdown of Incorrect Options:
HMG-CoA Synthase → Incorrect
HMG-CoA synthase is involved in the formation of HMG-CoA from acetyl-CoA , which occurs before the rate-limiting step .
It is not the main regulatory enzyme in cholesterol synthesis.
Mevalonate Carboxylase → Incorrect
This enzyme is involved in later steps of cholesterol synthesis , where mevalonate is converted into isoprenoid precursors.
It is not the rate-limiting step .
Prenyl Transferase → Incorrect
Prenyl transferase catalyzes the condensation of isoprenoid units in the cholesterol synthesis pathway.
It occurs after the rate-limiting step .
Squalene Synthase → Incorrect
Squalene synthase catalyzes the conversion of farnesyl pyrophosphate into squalene , an important step in late cholesterol synthesis .
It does not regulate the overall pathway .
Atropine is used in cases of severe bradycardia. What term describes its ability to modify the SA node?”
40 / 101
Category:
CVS – Pharmacology
A diabetic patient presents to the emergency department with a heart rate of 32 beats per minute and a blood sugar level of more than 500mg/dL. A drug ”atropine” is prescribed to increase the heart rate. What is this effect called?
Understanding the Effects of Atropine on Heart Rate
Atropine is a muscarinic antagonist that blocks the parasympathetic (vagal) influence on the heart, specifically at the SA node . This results in an increased heart rate , which is called a positive chronotropic effect .
Key Definitions:
Chronotropic effects → Affect heart rate
Positive chronotropic effect → Increases heart rate (e.g., atropine, epinephrine).
Negative chronotropic effect → Decreases heart rate (e.g., beta-blockers, acetylcholine).
Inotropic effects → Affect contractility (force of contraction)
Positive inotropic effect → Increases contractility (e.g., digoxin, epinephrine).
Negative inotropic effect → Decreases contractility (e.g., beta-blockers, calcium channel blockers).
Dromotropic effects → Affect conduction speed through the AV node
Positive dromotropic effect → Increases AV node conduction (e.g., epinephrine).
Negative dromotropic effect → Decreases AV node conduction (e.g., verapamil).
Breakdown of Incorrect Options:
Positive inotropic effect → Incorrect
Inotropic effects relate to contractility , not heart rate.
Atropine does not increase contractile force ; it only raises heart rate by inhibiting parasympathetic activity.
Negative chronotropic effect → Incorrect
A negative chronotropic effect means decreasing heart rate , which is the opposite of atropine’s action.
Drugs like beta-blockers and acetylcholine cause negative chronotropic effects.
Negative inotropic effect → Incorrect
Atropine does not weaken heart contraction ; it only affects heart rate.
Negative inotropes include beta-blockers and calcium channel blockers .
Negative dromotropic effect → Incorrect
A negative dromotropic effect slows AV node conduction , often seen with verapamil or beta-blockers .
Atropine does not slow conduction but instead increases heart rate via SA node stimulation.
“These capillaries have no pores, yet they need a way to transport nutrients and waste across their walls. How do they compensate for the lack of openings?”
41 / 101
Category:
CVS – Histology
In which of the blood vessels are pinocytotic vesicles seen?
Pinocytotic vesicles are specialized transport structures used for transcytosis , which allows the movement of macromolecules across the endothelial cells of certain blood vessels. These vesicles are abundant in continuous capillaries , which lack fenestrations and rely on vesicular transport to move substances between the blood and surrounding tissues.
Types of Capillaries and Their Characteristics:
Continuous Capillaries (Correct Answer)
Found in the muscle, skin, lungs, and brain (blood-brain barrier) .
Characterized by tight junctions between endothelial cells , no fenestrations , and a continuous basement membrane .
Pinocytotic vesicles facilitate transcytosis of fluids and small molecules across the endothelium.
Fenestrated Capillaries (Incorrect)
Found in the kidneys, intestines, and endocrine glands , where rapid exchange of small molecules is needed.
Contain fenestrations (small pores) that enhance permeability, reducing the need for pinocytotic vesicles.
Discontinuous Capillaries (Incorrect)
Found in the liver, spleen, and bone marrow .
Have large gaps between endothelial cells and a discontinuous basement membrane .
Permit free movement of large molecules and even cells , making vesicular transport unnecessary.
Sinusoids (Incorrect)
A subtype of discontinuous capillaries found in the liver, spleen, and bone marrow .
Extremely permeable due to large gaps between endothelial cells , negating the need for pinocytotic vesicles.
Aorta (Incorrect)
A large elastic artery with thick walls.
Does not rely on pinocytotic vesicles for transport since diffusion is minimal across its walls.
“Think about why muscle is heavier than fat. In biological structures, the denser component often provides structural support and function. Which macromolecule in lipoproteins serves this role?”
42 / 101
Category:
CVS – BioChemistry
The smallest lipoprotein particles of high-density lipoprotein cholesterol (HDL-C) are densest due to which of the following reasons?
The correct answer is: “Highest proportion of proteins to lipids.”
Understanding HDL and Lipoprotein Density
Lipoproteins are complexes of lipids and proteins that transport cholesterol and triglycerides in the blood. The density of a lipoprotein is determined by the ratio of proteins to lipids —since proteins are denser than lipids , a higher protein content increases the overall density.
Why is HDL the Densest Lipoprotein?
HDL has the highest proportion of proteins relative to lipids among all lipoproteins.
Unlike other lipoproteins (e.g., LDL, VLDL, chylomicrons) that are primarily composed of triglycerides and cholesterol esters, HDL is enriched with apolipoproteins (mainly ApoA-I and ApoA-II ).
The smallest HDL particles (HDL3 subtype) are particularly dense due to their very high protein content and minimal lipid content.
Breakdown of Incorrect Options:
“Minimal proportion of phosphate and highest proportion of lipids” → Incorrect
Lipids are less dense than proteins. A higher lipid content would make the lipoprotein less dense , not denser.
Phosphate groups are present in phospholipids, but their proportion does not determine density significantly.
“Lowest proportion of proteins to lipids” → Incorrect
This statement describes chylomicrons and VLDL , which have the lowest density because they contain mostly triglycerides.
“Highest proportion of phosphate and minimal proportion of lipids” → Incorrect
Phosphate groups are found in phospholipids, which are present in HDL, but density is primarily determined by the protein-to-lipid ratio , not phosphate content.
“Moderate proportion of proteins and highest proportion of lipids” → Incorrect
A high lipid content would decrease the density, making the lipoprotein larger and less dense, similar to LDL or VLDL.
It triggers a severe systemic immune response , causing widespread vasodilation, increased capillary permeability, and respiratory distress .
43 / 101
Category:
CVS – Physiology
A 20-year-old male presents to the emergency department with the complaint of an allergic reaction following peanuts consumption. Which of the following is most likely to be observed in this patient?
Why “Anaphylactic Shock” is the Correct Answer?
Anaphylactic shock is a life-threatening allergic reaction triggered by an allergen (e.g., peanuts, insect stings, medications).
It is IgE-mediated , leading to:
Mast cell degranulation → Release of histamine, bradykinin, and prostaglandins .
Widespread vasodilation → Hypotension and shock.
Increased capillary permeability → Fluid leakage into tissues, leading to edema.
Bronchoconstriction → Respiratory distress and wheezing.
Classic symptoms include:
Severe hypotension and shock
Flushed skin, urticaria (hives), and swelling
Difficulty breathing, wheezing, stridor, or anaphylactic airway obstruction
Immediate treatment:
Epinephrine (first-line treatment) to counteract vasodilation and bronchoconstriction
IV fluids to restore intravascular volume
Antihistamines and corticosteroids to prevent late-phase reactions
Why the Other Options Are Incorrect?
Neurogenic Shock – Incorrect
Neurogenic shock results from spinal cord injury , leading to loss of sympathetic tone and hypotension .
It is not caused by an allergic reaction .
Septic Shock – Incorrect
Septic shock is due to severe bacterial infection , leading to systemic vasodilation from inflammatory cytokines .
This patient has no signs of infection .
Cardiogenic Shock – Incorrect
Cardiogenic shock is due to heart failure (e.g., myocardial infarction, arrhythmia, or cardiac tamponade) .
This patient’s symptoms are due to an allergic reaction, not heart failure .
Hypovolemic Shock – Incorrect
Hypovolemic shock is due to fluid loss (e.g., hemorrhage, dehydration, burns) .
This patient has vasodilation and fluid redistribution, not volume loss .
“Which cardiac enzyme rises early after myocardial infarction but returns to normal within a few days, making it useful for detecting reinfarction?”
44 / 101
Category:
CVS – BioChemistry
A 48-year-old man presents to the emergency department with a complaint of chest pain for the past 4-5 hours. He is profusely sweating and also has slight dyspnea. Which of the following is expected to be elevated?
Understanding Cardiac Biomarkers in Myocardial Infarction (MI)
When cardiac myocytes undergo ischemic damage (as in acute myocardial infarction, MI ), they release cardiac-specific enzymes and proteins into the bloodstream.
CK-MB as a Biomarker for MI:
Creatine kinase-MB (CK-MB) is an enzyme found in cardiac muscle cells.
It is released into the bloodstream after myocardial injury , making it a valuable early marker for myocardial infarction.
Timeline of CK-MB Release in MI:
Biomarker
Rise Time
Peak Level
Return to Normal
CK-MB
3–6 hours
12–24 hours
48–72 hours
Troponin I/T
3–4 hours
24–48 hours
7–10 days
Lactate Dehydrogenase (LDH)
24–48 hours
3–6 days
7–14 days
Since this patient presented with chest pain for 4–5 hours , CK-MB would be elevated in his blood.
Troponin I/T is a more specific marker for MI , but CK-MB remains useful for detecting reinfarction due to its quicker return to baseline.
Breakdown of Incorrect Options:
Pyruvate Kinase → Incorrect
Pyruvate kinase is an enzyme involved in glycolysis and is not specific to cardiac muscle .
It does not serve as a diagnostic marker for myocardial infarction.
Hexokinase → Incorrect
Hexokinase catalyzes glucose phosphorylation in glycolysis.
It is not released in myocardial infarction and has no diagnostic relevance for cardiac damage .
Alkaline Phosphatase (ALP) → Incorrect
ALP is primarily a marker for bone and liver disease .
It is not associated with myocardial infarction.
Lactate Dehydrogenase (LDH) → Incorrect
LDH is elevated in MI but rises later (24–48 hours after infarction) .
Since the patient’s symptoms started 4–5 hours ago , LDH would not be significantly elevated yet .
LDH is also less specific than CK-MB or troponin .
“The artery most commonly involved in an inferior MI supplies both the right ventricle and part of the left ventricle. Think of which vessel dominates the posterior circulation of the heart in most individuals.”
45 / 101
Category:
CVS – Physiology
A 50-year-old patient presented to the emergency department with a myocardial infarction of the inferior wall of the left ventricle. Which of the following arteries is most likely to be damaged?
A myocardial infarction (MI) of the inferior wall of the left ventricle is most commonly caused by an occlusion of the right coronary artery (RCA) . The inferior wall of the left ventricle receives its blood supply primarily from the posterior descending artery (PDA) , which in right-dominant circulation (85% of individuals) arises from the RCA .
Key Anatomical Considerations:
Right Coronary Artery (RCA)
In right-dominant individuals (majority of the population) , the RCA supplies:
The inferior wall of the left ventricle .
The posterior part of the interventricular septum .
The right atrium and right ventricle .
The sinoatrial (SA) node (60% of cases) and atrioventricular (AV) node (80% of cases) .
A blockage in the RCA can lead to inferior wall MI , often seen as ST-segment elevations in leads II, III, and aVF on ECG .
Left Circumflex Artery (LCX) (Incorrect)
The LCX supplies the lateral wall of the left ventricle and sometimes the posterior wall in left-dominant circulation (15% of individuals) .
It does not typically supply the inferior wall in right-dominant individuals.
Left Anterior Descending Artery (LAD) (Incorrect)
The LAD supplies the anterior wall of the left ventricle and the anterior two-thirds of the interventricular septum .
Occlusion of the LAD leads to an anterior wall MI , not an inferior wall MI .
Aorta (Incorrect)
The aorta gives rise to the coronary arteries but is not directly responsible for the supply of the inferior wall of the left ventricle.
Pulmonary Artery (Incorrect)
The pulmonary artery carries deoxygenated blood from the right ventricle to the lungs .
It plays no role in myocardial blood supply .
Clinical Correlation:
Inferior wall MI (RCA occlusion) can be associated with:
Bradycardia and hypotension due to involvement of the SA node and AV node .
Right ventricular infarction , leading to jugular venous distension (JVD) and clear lungs on auscultation.
ST-segment elevations in leads II, III, and aVF on ECG.
“Think about the balance between cost, availability, and the information needed for an initial evaluation. Which test provides a quick, affordable, and effective assessment of heart size and chamber enlargement without requiring advanced technology?”
46 / 101
Category:
CVS – Radiology
A young man undergoing a pre-recruitment check-up is suspected of cardiomegaly on examination. Which of the following is the most appropriate and cost-effective radiological investigation for determining the size of the heart and the chamber responsible for the enlargement?
Why Chest X-ray (Posteroanterior view) is Correct:
A chest X-ray (posteroanterior view) is the most appropriate and cost-effective initial investigation for evaluating cardiomegaly (enlargement of the heart). It provides a clear image of the heart’s silhouette and can help determine the overall size of the heart and identify specific chamber enlargement based on the shape and contours of the cardiac shadow.
Cost-effectiveness: Chest X-ray is widely available, inexpensive, and requires minimal resources compared to advanced imaging techniques.
Utility: It is a first-line screening tool for cardiomegaly and can also provide additional information about the lungs and thoracic structures, which may be relevant in the context of cardiac enlargement.
Why the Other Options Are Incorrect:
Nuclear Scan:
Nuclear scans (e.g., myocardial perfusion imaging) are used to assess blood flow to the heart muscle and diagnose ischemic heart disease. They are not suitable for evaluating the size of the heart or identifying specific chamber enlargement. Additionally, they are expensive and involve radiation exposure.
Contrast-enhanced Computerized Tomography (CT) Scan:
While CT scans can provide detailed images of the heart and its chambers, they are not cost-effective for initial evaluation of cardiomegaly. CT scans are typically reserved for specific indications, such as evaluating coronary arteries or complex congenital heart disease.
Cardiac Magnetic Resonance Imaging (MRI):
Cardiac MRI provides highly detailed images of the heart’s structure and function, including chamber size and wall motion. However, it is expensive, time-consuming, and not widely available. It is typically used for complex cases or when other imaging modalities are inconclusive.
Doppler Echocardiography:
Doppler echocardiography is an excellent tool for evaluating heart size, chamber dimensions, and function. However, it is not a radiological investigation (it uses ultrasound) and is typically performed after initial screening with a chest X-ray. While highly informative, it is more specialized and may not be as cost-effective as a chest X-ray for initial screening.
“Which blood vessel must withstand the highest pressure fluctuations and store energy during systole to ensure continuous blood flow during diastole?”
47 / 101
Category:
CVS – Histology
The microscopic structure of the tunica media consists of concentric layers of smooth muscle fibers with variable amounts of reticular and elastic fibers in between. Which of the following vessels contains the largest percentage of elastic fibers in their tunica media?
Understanding the Composition of the Tunica Media in Different Vessels
The tunica media is the middle layer of a blood vessel wall, primarily composed of smooth muscle cells , elastic fibers , and collagen . The proportion of elastic fibers varies depending on the type of vessel and its function.
Why Does the Aorta Have the Most Elastic Fibers?
The aorta is an elastic artery , meaning its tunica media contains the highest percentage of elastic fibers compared to any other blood vessel.
These elastic fibers allow the aorta to stretch and recoil in response to pulsatile blood flow from the heart.
This elasticity helps maintain continuous blood flow during diastole by storing energy from systolic contraction and releasing it when the heart relaxes.
Breakdown of Incorrect Options:
Jugular Vein → Incorrect
Veins have thinner walls and do not require as many elastic fibers as arteries.
The jugular vein primarily relies on valves and external forces (e.g., skeletal muscle contractions) to facilitate blood return to the heart , rather than elasticity.
Inferior Vena Cava (IVC) → Incorrect
The IVC is a large vein , but veins generally contain very few elastic fibers compared to arteries.
The IVC has a thin tunica media with more collagen fibers to provide structural support rather than elasticity.
Coronary Artery → Incorrect
The coronary arteries are muscular arteries , meaning they have a higher proportion of smooth muscle and fewer elastic fibers compared to the aorta.
Their primary role is to regulate blood flow through vasoconstriction and vasodilation , not to accommodate high-pressure pulsations like the aorta.
Medium-Sized Artery → Incorrect
Medium-sized arteries (also called muscular arteries ) contain some elastic fibers , but they have more smooth muscle than elastic tissue .
These arteries distribute blood to organs and adjust blood flow through vasomotion , unlike the aorta, which must expand and recoil with each heartbeat.
Baroreceptors are pressure-sensitive stretch receptors located in the carotid sinus and aortic arch . When they are excited (stimulated by high blood pressure) , what kind of reflex response occurs to restore homeostasis
48 / 101
Category:
CVS – Physiology
Which of the following effects results from the excitation of baroreceptors in arteries?
Baroreceptors play a key role in short-term blood pressure regulation by detecting changes in arterial pressure and triggering autonomic responses via the baroreflex .
When arterial pressure increases , baroreceptors in the carotid sinus and aortic arch are stretched and stimulated .
This results in increased firing of afferent signals via the glossopharyngeal (CN IX) and vagus (CN X) nerves to the nucleus tractus solitarius (NTS) in the medulla .
The parasympathetic (vagal) activity is enhanced , while sympathetic outflow is inhibited .
The net effect is a reduction in blood pressure and heart rate to maintain homeostasis.
Why “Decreased Arterial Pressure” is the Correct Answer:
Increased baroreceptor activity leads to reflex bradycardia and vasodilation, reducing arterial pressure.
Parasympathetic activation decreases heart rate and cardiac output .
Sympathetic inhibition leads to reduced vascular resistance , further lowering blood pressure.
This is part of the baroreflex mechanism , which helps prevent excessive increases in blood pressure.
Why the Other Options Are Incorrect:
Peripheral Vasoconstriction – Incorrect
Baroreceptor stimulation inhibits the sympathetic nervous system, causing vasodilation , not vasoconstriction.
Vasoconstriction occurs when baroreceptors are suppressed (e.g., during hypotension), not when they are excited.
Vasovagal Syncope – Incorrect
Vasovagal syncope (fainting) occurs due to sudden excessive vagal activation , leading to severe bradycardia and vasodilation .
While baroreceptors play a role in autonomic regulation, normal baroreceptor excitation does not cause syncope —vasovagal syncope is triggered by strong emotional stress, pain, or standing too long.
Prolonged PR Interval – Incorrect
The PR interval represents AV node conduction time .
While increased vagal tone can slightly slow AV conduction , normal baroreceptor activation does not significantly prolong the PR interval .
Increased Cardiac Output – Incorrect
Baroreceptor excitation leads to decreased cardiac output, not an increase.
The parasympathetic system reduces heart rate , which lowers stroke volume and cardiac output .
“The right atrium has both smooth and rough parts. Which structure, derived from an embryonic venous cavity, forms the smooth portion where major veins enter?”
49 / 101
Category:
CVS – Embryology
A smooth part in the right atrium, which is a remnant of the right sinus horn is seen by some medical students while dissecting a cadaver. The part can be identified as which of the following?
Understanding the Sinus Venarum and Its Embryological Origin
The sinus venarum is the smooth-walled part of the right atrium , which is derived from the right horn of the sinus venosus during embryonic development.
Key Features of the Sinus Venarum:
It forms the posterior smooth portion of the right atrium.
It is separate from the rough anterior portion (pectinate muscles) by the crista terminalis .
It serves as the entry site for the superior and inferior vena cava , allowing venous return to the right atrium.
Breakdown of Incorrect Options:
Crista Terminalis → Incorrect
The crista terminalis is a ridge of muscle that separates the smooth sinus venarum from the rough pectinate muscles .
It is not derived from the right sinus horn , but rather marks the boundary between embryonic sinus venosus and primitive atrium.
Coronary Sinus → Incorrect
The coronary sinus is the main venous drainage of the heart , but it is derived from the left sinus horn , not the right sinus horn.
It opens into the right atrium near the tricuspid valve .
Fossa Ovalis → Incorrect
The fossa ovalis is the remnant of the foramen ovale , a fetal structure that allowed blood to bypass the lungs.
It is located in the interatrial septum , not the right atrial wall .
Right Ventricle → Incorrect
The right ventricle is a different chamber of the heart and has no embryological connection to the right sinus horn .
It pumps blood to the pulmonary circulation , while the sinus venarum is part of the right atrium .
“Which layer of the heart is composed of contractile cells that actively generate the force required to push blood forward?”
50 / 101
Category:
CVS – Histology
Which of the following is involved in generating the pumping force of the heart?
Understanding the Myocardium’s Role in Pumping the Heart
The myocardium is the muscular layer of the heart responsible for generating the force required for contraction and blood ejection . It is composed of cardiac muscle fibers that work in a coordinated manner to pump blood throughout the body.
Why is the Myocardium Responsible for Pumping Force?
Contractile Function:
The myocardium contains cardiac myocytes that generate the contractile force needed to propel blood from the heart into circulation.
Excitation-Contraction Coupling:
It responds to electrical impulses from the cardiac conduction system (SA node → AV node → Purkinje fibers) to produce coordinated contractions .
Thickest Layer in the Ventricles:
The left ventricle has the thickest myocardium to generate high pressure required to pump blood into systemic circulation.
The right ventricle has a thinner myocardium , as it only needs to pump blood to the lungs.
Breakdown of Incorrect Options:
Papillary Muscles → Incorrect
Papillary muscles are part of the myocardium , but their primary function is to anchor the chordae tendineae , preventing valve prolapse during contraction.
They do not generate the main pumping force of the heart.
Endothelium → Incorrect
The endothelium is a thin layer of cells lining the blood vessels and heart chambers .
It plays a role in vascular function and preventing clot formation , but it does not contribute to contraction or pumping force .
Epicardium → Incorrect
The epicardium is the outermost layer of the heart, composed of connective tissue and fat.
It provides structural support and lubrication (via the pericardial fluid) but does not participate in contraction .
Endocardium → Incorrect
The endocardium is the innermost layer of the heart, providing a smooth surface to prevent blood clot formation.
It does not contribute to the mechanical pumping action of the heart.
“During cardiac surgery, a surgeon may need to isolate the major arteries before connecting a heart-lung machine. Which pericardial sinus allows this maneuver?”
51 / 101
Category:
CVS – Anatomy
During cardiac surgery, the surgeon passes his finger through a potential space, that separates the great arteries from the atria and the veins. Which of the following potential spaces is it?
Understanding the Transverse Pericardial Sinus
The transverse pericardial sinus is a potential space within the pericardial cavity that separates the great arteries (aorta and pulmonary trunk) from the great veins (superior vena cava, left atrium, and pulmonary veins).
It is located posterior to the ascending aorta and pulmonary trunk and anterior to the superior vena cava and atria .
This space is particularly important in cardiac surgery , as surgeons can pass a finger or a surgical clamp through it to control or isolate the great arteries (aorta and pulmonary trunk) during procedures like cardiopulmonary bypass .
Breakdown of Incorrect Options:
Oblique Pericardial Sinus → Incorrect
The oblique pericardial sinus is a blind-ended space located behind the left atrium , between the pulmonary veins .
It does not separate the great arteries from the veins .
Surgeons do not commonly pass a finger through it during cardiac surgery.
Anterior Costomediastinal Recess → Incorrect
This is a pleural recess , part of the lungs’ pleural cavity , not the pericardial cavity.
It does not involve the heart or the great vessels.
Costodiaphragmatic Recess → Incorrect
This is another pleural recess , where the costal pleura meets the diaphragmatic pleura .
It allows for lung expansion , but it has no relation to the pericardial sinuses .
Superior Aortic Recess → Incorrect
This is an imprecise or non-standard term in clinical anatomy.
The superior mediastinum contains the aortic arch, but there is no recognized “superior aortic recess” as a distinct anatomical space .
“The electrical impulse must first excite the atria before reaching the ventricles. Think about which structure naturally sets the rhythm and which one ensures a delay before the impulse moves downward.”
52 / 101
Category:
CVS – Physiology
What is the correct sequence of the conduction of cardiac impulses in the heart?
The cardiac conduction system ensures the heart contracts in a coordinated manner to pump blood efficiently. The sequence of impulse conduction is crucial for synchronized contraction of the atria and ventricles.
Step-by-Step Breakdown of the Correct Sequence:
Sinoatrial (SA) Node:
The SA node , located in the right atrium near the superior vena cava, is the natural pacemaker of the heart .
It generates impulses at a rate of 60-100 beats per minute in a normal individual.
The impulse spreads through the atria, causing atrial contraction .
Atrioventricular (AV) Node:
The impulse reaches the AV node , located at the junction of the atria and ventricles (interatrial septum, near the tricuspid valve).
The AV node delays the impulse (~0.1 sec) to allow the ventricles to fill with blood before contraction.
Bundle of His:
The impulse then travels through the bundle of His , which is located in the interventricular septum .
This structure serves as the only electrical connection between the atria and ventricles.
Left and Right Bundle Branches:
The bundle of His splits into the left and right bundle branches , which run along either side of the interventricular septum.
The right bundle branch conducts impulses to the right ventricle , while the left bundle branch supplies the left ventricle .
Purkinje Fibers:
Finally, the Purkinje fibers distribute the impulse rapidly throughout the ventricular myocardium, ensuring a coordinated and forceful ventricular contraction .
Why the Other Options Are Incorrect:
SA node → bundle of His → Purkinje fibers → AV node → left and right bundle branches (Incorrect)
The AV node must come before the bundle of His.
The Purkinje fibers activate the ventricles , so placing them before the AV node is incorrect.
SA node → AV node → Purkinje fibers → bundle of His → left and right bundle branches (Incorrect)
The bundle of His comes before the Purkinje fibers, not after.
Purkinje fibers are the final step before ventricular contraction.
SA node → bundle of His → AV node → Purkinje fibers → left and right bundle branches (Incorrect)
The AV node delays the impulse before it reaches the bundle of His.
The bundle of His does not receive impulses before the AV node.
SA node → Purkinje fibers → AV node → bundle of His → left and right bundle branches (Incorrect)
The Purkinje fibers are the last step , so they cannot come before the AV node.
“The heart needs a ‘pause button’ to ensure the ventricles don’t contract before they’re fully filled. Which structure provides this crucial delay?”
53 / 101
Category:
CVS – Physiology
Which part of the heart causes a delay in impulse transmission?
Understanding the AV Node and the Delay in Impulse Transmission
The AV node is responsible for delaying the transmission of electrical impulses from the atria to the ventricles . This delay is crucial for proper cardiac function because it ensures that the atria have enough time to fully contract and pump blood into the ventricles before ventricular contraction begins.
Why Does the AV Node Cause a Delay?
Slow Conduction Fibers:
The AV node contains fewer gap junctions , leading to slower electrical signal propagation.
Physiological Purpose of the Delay:
Allows the atria to completely empty blood into the ventricles before the ventricles contract.
Prevents excessively fast transmission of impulses, which could cause ventricular fibrillation if the ventricles contract too quickly.
Autonomic Regulation:
Parasympathetic stimulation (via the vagus nerve) prolongs the AV delay (negative dromotropic effect).
Sympathetic stimulation shortens the delay, allowing a faster heart rate when needed.
Breakdown of Incorrect Options:
Sinoatrial (SA) Node → Incorrect
The SA node is the pacemaker of the heart, initiating impulses.
However, it does not delay conduction —it generates impulses at the fastest rate (60–100 beats per minute).
Internodal Fibers → Incorrect
These fibers rapidly transmit impulses from the SA node to the AV node , with minimal delay.
Purkinje Fibers → Incorrect
Purkinje fibers have the fastest conduction velocity (2–4 m/s) to ensure rapid and synchronized ventricular contraction.
They do not cause any delay ; instead, they speed up impulse transmission .
Bundle Branches → Incorrect
The left and right bundle branches conduct impulses from the AV node to the Purkinje fibers quickly.
They do not slow conduction.
During the cardiac action potential , different ions flow across the cardiac muscle cell membrane at different phases. Potassium (K⁺) is responsible for repolarization and resting membrane potential maintenance . In which phase does K⁺ permeability peak , allowing the cell to return to its resting state?
54 / 101
Category:
CVS – Physiology
In which of the following phases of the cardiac muscle action potential, potassium ions permeability is the highest?
Why “Phase 3” is the Correct Answer?
Phase 3 (Repolarization) is when K⁺ conductance is at its peak.
During this phase, voltage-gated K⁺ channels (delayed rectifier K⁺ channels) open, leading to a massive K⁺ efflux out of the cell .
This restores the resting membrane potential by making the intracellular environment more negative.
Calcium (Ca²⁺) influx stops , and the dominance of K⁺ outflow leads to full repolarization.
Why the Other Options Are Incorrect?
Phase 0 (Depolarization) – Incorrect
Phase 0 is due to a rapid Na⁺ influx through voltage-gated sodium channels .
K⁺ channels are mostly closed in this phase, meaning low K⁺ permeability .
Phase 1 (Initial Repolarization) – Incorrect
A small transient outward K⁺ current occurs here, but K⁺ permeability is not at its peak .
Sodium channels inactivate , and potassium briefly exits the cell before the plateau phase.
Phase 2 (Plateau Phase) – Incorrect
The plateau phase results from a balance between inward Ca²⁺ influx (L-type calcium channels) and outward K⁺ efflux .
K⁺ permeability is increased but not at its peak.
Phase 4 (Resting Membrane Potential) – Incorrect
K⁺ permeability is moderate at rest, maintaining a stable negative resting potential.
However, it is not as high as in Phase 3 , where repolarization is actively occurring.
“In moments of crisis, people often need to feel heard more than they need immediate solutions. What communication technique reassures them that their message is being understood without changing its meaning?”
55 / 101
Category:
CVS – Community Medicine/ Behavioural Sciences
What does the communication strategy in crisis intervention known as “paraphrasing” entail?
Concept Breakdown:
In crisis intervention and active listening , paraphrasing is a key communication strategy used to demonstrate understanding and empathy. It involves restating the essence of what the person has said in your own words to show that you are actively listening and comprehending their message.
This technique does not mean simply repeating the exact words but rather summarizing the key idea in a way that shows empathy and engagement .
Why the Other Options Are Incorrect:
Asking questions to clarify a person’s statement – Incorrect
This refers to clarification , which involves directly asking the person to explain or elaborate rather than restating their thoughts .
Example: “Can you tell me more about what’s making you feel overwhelmed?”
Using non-verbal communication such as maintaining eye contact, head nodding, etc. – Incorrect
These are non-verbal active listening techniques , which support communication but are not paraphrasing .
They indicate attentiveness but do not verbally reflect back the person’s thoughts .
Helping the person identify and articulate emotions – Incorrect
This describes emotional labeling or reflection of feelings , where the listener helps the speaker name their emotions (e.g., “You sound frustrated.” ).
Paraphrasing, on the other hand, focuses on repeating and rewording content, not necessarily emotions .
Summarizing multiple elements of a person’s message/statement – Incorrect
Summarizing involves condensing a conversation into key takeaways , which is broader than paraphrasing.
Example: “So, from what you’ve told me, you’re feeling overwhelmed because of stress at work and family issues.”
Paraphrasing is shorter and more immediate , often focusing on a single statement rather than the whole conversation.
The coronary sinus is the largest cardiac vein and is responsible for draining most of the venous blood from the heart into the right atrium . Consider where it lies.. serving as a pathway to both coronary arteries and veins
56 / 101
“This large vein carries blood from the upper body back to the heart. It drains into the chamber that acts as the entry point for deoxygenated blood.”
57 / 101
Category:
CVS – Anatomy
Which of the following provides an afferent blood supply to the right atrium?
The right atrium is responsible for receiving deoxygenated blood from the systemic circulation and directing it into the right ventricle, which then pumps it to the lungs for oxygenation.
Sources of Afferent Blood Supply to the Right Atrium:
Superior Vena Cava (SVC) – Brings deoxygenated blood from the upper body (head, neck, upper limbs, and thorax) .
Inferior Vena Cava (IVC) – Brings deoxygenated blood from the lower body (abdomen, pelvis, and lower limbs) .
Coronary Sinus – Returns deoxygenated blood from the myocardium (heart muscle) itself .
Thus, among the given choices, the superior vena cava is the correct answer as it is one of the main afferent vessels supplying blood to the right atrium .
Why the Other Options Are Incorrect:
Aorta (Incorrect)
The aorta carries oxygenated blood from the left ventricle to the systemic circulation.
It does not supply blood to the right atrium.
Coronary Arteries (Incorrect)
Coronary arteries arise from the aorta and supply oxygenated blood to the heart muscle itself , not the right atrium.
However, the coronary sinus , which collects venous blood from the heart, does drain into the right atrium.
Pulmonary Veins (Incorrect)
Pulmonary veins bring oxygenated blood from the lungs to the left atrium , not the right atrium.
They play a role in systemic circulation , not venous return to the right atrium.
Pulmonary Arteries (Incorrect)
Pulmonary arteries carry deoxygenated blood from the right ventricle to the lungs .
They are part of the pulmonary circulation , not the systemic venous return to the right atrium.
Heart disease is strongly linked to cholesterol levels, inflammation, and blood clot formation. Think about which type of fat has the best evidence for reducing ‘bad’ cholesterol (LDL), improving ‘good’ cholesterol (HDL), and protecting blood vessels from inflammation and damage.”
58 / 101
Category:
CVS – BioChemistry
Consumption of which class of dietary fatty acids substantially relates with the lowering of the risk of developing heart disease?
Dietary fats play a crucial role in cardiovascular health, and their composition significantly influences lipid metabolism, inflammation, and overall heart disease risk. Among the different types of dietary fatty acids, polyunsaturated fatty acids (PUFAs) have been strongly linked to reducing the risk of cardiovascular disease (CVD) due to their beneficial effects on lipid profiles, endothelial function, and inflammation.
Thus, PUFAs are the most effective dietary fatty acids for reducing heart disease risk .
Why the Other Options Are Incorrect:
Saturated fatty acids – Incorrect
Found in butter, red meat, full-fat dairy, and palm oil .
Increase LDL cholesterol , which contributes to atherosclerosis and heart disease.
Excess consumption is linked to higher cardiovascular risk .
Unsaturated fatty acids – Incorrect
This is a broad term that includes both monounsaturated (MUFAs) and polyunsaturated (PUFAs) .
While both MUFAs and PUFAs have heart benefits, PUFAs have stronger evidence in lowering heart disease risk.
Monounsaturated fatty acids (MUFAs) – Incorrect
Found in olive oil, avocados, and nuts .
Have cardiovascular benefits , such as reducing LDL cholesterol and increasing HDL cholesterol .
While beneficial, PUFAs have a stronger association with lowering heart disease risk compared to MUFAs.
Medium saturated fatty acids – Incorrect
Likely referring to medium-chain triglycerides (MCTs) found in coconut oil and dairy .
MCTs are metabolized differently (rapid energy source) but do not significantly reduce heart disease risk like PUFAs do.
Some studies suggest neutral or even increased cardiovascular risk with high intake.
Shock occurs when there is inadequate perfusion of tissues , leading to organ dysfunction . A spinal injury can disrupt autonomic nervous system control , particularly sympathetic outflow , causing severe vasodilation and hypotension . Which type of shock is characterized by loss of vascular tone due to nervous system damage ?
59 / 101
Category:
CVS – Pathology
Which of the following is a type of shock that results from a spinal injury?
Why “Neurogenic Shock” is the Correct Answer?
Neurogenic shock occurs due to a spinal cord injury, especially above T6 , leading to:
Loss of sympathetic tone → Widespread vasodilation
Bradycardia (unlike other types of shock, which cause tachycardia)
Severe hypotension due to pooling of blood in peripheral vessels
Warm, flushed skin (instead of cool, clammy skin seen in other types of shock)
Common causes of neurogenic shock:
Traumatic spinal cord injury (above T6)
Spinal anesthesia
Brainstem injuries affecting autonomic control
Why the Other Options Are Incorrect?
Anaphylactic Shock – Incorrect
Anaphylactic shock is caused by severe allergic reactions , leading to histamine release and systemic vasodilation .
It is due to immune system overactivation , not a spinal injury.
Cardiogenic Shock – Incorrect
Cardiogenic shock results from heart failure (e.g., myocardial infarction, arrhythmias, or cardiac tamponade) , leading to reduced cardiac output .
Spinal cord injury does not directly impair cardiac pump function .
Septic Shock – Incorrect
Septic shock is caused by severe infections , leading to systemic inflammation, vasodilation, and hypotension .
It is due to bacterial toxins , not a spinal cord injury.
Hypovolemic Shock – Incorrect
Hypovolemic shock occurs due to severe fluid loss (e.g., hemorrhage, dehydration, burns) , leading to low blood volume and decreased perfusion .
Spinal cord injury does not cause major fluid loss , so it does not fit this category.
“Which fetal shunt responds to rising oxygen and falling prostaglandins, typically closing to separate the pulmonary and systemic circulations?”
60 / 101
Category:
CVS – Embryology
Prenatally, the ductus arteriosus shunts blood from the pulmonary trunk to the descending aorta. By what time is it closed postnatally in full-term infants?
Closure of the Ductus Arteriosus in Full-Term Infants
Functional Closure (Within 48 Hours):
At birth, the baby takes its first breath , increasing oxygen levels and decreasing prostaglandin E₂ (PGE₂) levels.
This causes smooth muscle contraction in the ductus arteriosus, leading to gradual functional closure within 48 hours in most full-term infants.
Some constriction begins within 24 hours , but complete functional closure typically takes up to 48 hours .
Anatomical Closure (Weeks to Complete):
After functional closure, the ductus arteriosus undergoes fibrosis , forming the ligamentum arteriosum .
This permanent anatomical closure takes about 1–3 weeks .
Breakdown of Incorrect Options:
12 Hours → Incorrect
The ductus may begin constricting, but it does not fully close functionally this quickly in most cases.
24 Hours → Incorrect
Many full-term infants show significant ductal narrowing at 24 hours, but closure is usually complete by 48 hours .
72 Hours → Incorrect
While closure is expected by 48 hours , persistent ductus arteriosus beyond 72 hours may indicate delayed closure or PDA (patent ductus arteriosus) .
96 Hours → Incorrect
By 96 hours (4 days), functional closure should have already occurred in full-term infants . If it remains open, it is considered pathological (PDA) .
“This chamber is a gateway for oxygenated blood before it gets pumped into the body. It must pass through a one-way valve before entering the heart’s most powerful pumping chamber.”
61 / 101
Category:
CVS – Anatomy
Which of the following structures does the left atrium open into?
The left atrium is one of the four chambers of the heart, responsible for receiving oxygenated blood from the pulmonary veins and delivering it to the left ventricle , which then pumps it into the systemic circulation.
Step-by-Step Breakdown of Blood Flow:
Oxygenated blood from the lungs enters the left atrium through the pulmonary veins .
When the left atrium contracts, it pushes the blood through the mitral (bicuspid) valve into the left ventricle .
The left ventricle then pumps the oxygenated blood into the aorta , which distributes it to the rest of the body.
Why the Other Options Are Incorrect:
Right Atrium (Incorrect)
The right atrium receives deoxygenated blood from the superior and inferior vena cava , not from the left atrium .
The left atrium does not open into the right atrium.
Right Ventricle (Incorrect)
The right ventricle is part of the pulmonary circulation , pumping deoxygenated blood to the lungs via the pulmonary arteries .
The left atrium does not connect to the right ventricle .
Pulmonary Arteries (Incorrect)
Pulmonary arteries carry deoxygenated blood from the right ventricle to the lungs .
The left atrium receives blood from the pulmonary veins but does not open into the pulmonary arteries.
Pulmonary Veins (Incorrect)
The pulmonary veins bring oxygenated blood into the left atrium , but the left atrium does not open into them .
Instead, it acts as a receiving chamber before sending blood to the left ventricle .
“Consider the urgency of the situation and the need for both diagnosis and immediate treatment. Which test not only identifies the problem but also allows for intervention to restore blood flow to the heart?”
62 / 101
Category:
CVS – Radiology
A 45-year-old woman presents to the outpatient department with a complaint of typical chest pain radiating to her left arm associated with vomiting for the past three hours. She has a habit of smoking. Her vitals show a blood pressure of 130/70 mmHg and a regular pulse of 130 beats per minute. The systemic examination is remarkable. Her electrocardiogram(ECG) showed an ST-segment elevation in inferior ECG leads. What would be the most appropriate test for this patient?
Why Coronary Angiography is Correct:
The patient presents with typical symptoms of acute coronary syndrome (ACS) , including chest pain radiating to the left arm, vomiting, and ST-segment elevation in the inferior ECG leads (II, III, and aVF). These findings are highly suggestive of an ST-elevation myocardial infarction (STEMI) , which is a medical emergency requiring immediate intervention.
Coronary angiography is the most appropriate test in this scenario because it provides direct visualization of the coronary arteries, identifies the location and severity of the blockage, and allows for immediate therapeutic intervention (e.g., percutaneous coronary intervention or PCI) to restore blood flow. Coronary angiography is the gold standard for diagnosing and managing STEMI.
Why the Other Options Are Incorrect:
Nuclear Imaging:
Nuclear imaging (e.g., myocardial perfusion scans) is used to assess blood flow to the heart muscle and diagnose ischemic heart disease. However, it is not appropriate in the acute setting of STEMI, where immediate coronary angiography is required for definitive diagnosis and treatment.
Exercise Tolerance Test:
This test is used to evaluate for coronary artery disease in stable patients with atypical symptoms or risk factors. It is contraindicated in patients with acute chest pain and ST-segment elevation, as it could worsen the condition.
Trans-thoracic Echocardiography (TTE):
TTE is useful for assessing cardiac structure and function, such as wall motion abnormalities or complications of myocardial infarction (e.g., mitral regurgitation or ventricular septal defect). However, it does not provide direct information about coronary artery blockages and is not the first-line test for STEMI.
Dobutamine Stress Echocardiography:
This test is used to evaluate for ischemic heart disease in stable patients or assess myocardial viability. It is not appropriate in the acute setting of STEMI, where immediate coronary angiography is indicated.
“Since the heart spends more time in diastole than systole, why does the MAP formula give greater weight to diastolic pressure?”
63 / 101
Category:
CVS – Physiology
What will be the mean arterial pressure of a person having a systolic blood pressure of 130 mmHg and a diastolic blood pressure of 70 mmHg?
Calculation of Mean Arterial Pressure (MAP)
The mean arterial pressure (MAP) is the average blood pressure in a person’s arteries during one cardiac cycle . It is calculated using the formula:
MAP = Diastolic BP + 1/3 (Systolic BP – Diastolic BP)
Given:
Systolic BP (SBP) = 130 mmHg
Diastolic BP (DBP) = 70 mmHg
MAP = 70 + 1/3 (130-70)
MAP = 70 + 1/3(60)
MAP = 70 + 20 = 90 mmHg
Breakdown of Incorrect Options:
80 mmHg → Incorrect
This would be an incorrect calculation of MAP.
95 mmHg → Incorrect
Slightly overestimated; MAP must be calculated with the correct formula.
75 mmHg → Incorrect
This underestimates the MAP.
100 mmHg → Incorrect
Overestimated; MAP is lower than this.
Atherosclerosis preferentially affects large and medium-sized arteries , especially at sites of turbulent blood flow and shear stress . Consider which anatomical locations experience the highest mechanical stress and endothelial injury , making them more susceptible to plaque formation.
64 / 101
Category:
CVS – Pathology
Which of the following blood vessels is the most prone to atherosclerosis?
The abdominal aorta (especially below the renal arteries) is one of the most common locations for atherosclerotic plaque formation .
Why the “Anterior Wall of the Abdominal Aorta” is a Strong Answer:
High Mechanical Stress: The abdominal aorta is subject to higher hemodynamic stress due to the pulsatile nature of blood flow.
Lack of Vasa Vasorum: Unlike the thoracic aorta, the abdominal aorta has fewer vasa vasorum (small vessels that supply large arteries) , making it more prone to ischemic damage and atherosclerosis.
Turbulent Blood Flow: The abdominal aorta experiences flow disturbances, especially near bifurcations , increasing endothelial damage and lipid deposition.
Common Atherosclerotic Site: The infrarenal abdominal aorta (below the renal arteries) is a well-documented location for atherosclerosis due to blood flow patterns and wall stress.
Why “At the Point of Bifurcation of Arteries” is Also a Valid Answer:
Atherosclerosis is most severe at arterial bifurcations because they experience turbulent flow and shear stress , leading to endothelial dysfunction and plaque deposition .
Common bifurcation sites for atherosclerosis:
Carotid bifurcation → Can lead to stroke.
Aortic bifurcation (iliac arteries) → Common site for plaque formation.
Coronary artery bifurcations → Increases risk of myocardial infarction.
Why the Other Options Are Incorrect:
Radial Artery – Incorrect
The radial artery is a small muscular artery with high blood flow velocity and low turbulence , making it less prone to atherosclerosis .
Atherosclerosis primarily affects large and medium-sized elastic arteries , not small muscular arteries.
Hepatic Arteries – Incorrect
The hepatic arteries have a high perfusion rate and are rarely affected by atherosclerosis.
The portal circulation provides collateral blood supply , reducing the impact of plaque formation.
Renal Arteries – Incorrect
While renal artery stenosis can occur due to atherosclerosis , it is not the most common site for plaque development.
The proximal renal artery may be affected, but major plaques are more frequent at larger arterial bifurcations .
Think about how the body responds to transient ischemia and how mast cells in the myocardium might contribute to pain sensation. What mediator is commonly associated with both allergic reactions and vasodilation, yet can also be involved in localized ischemic responses?
65 / 101
Category:
CVS – Pathology
A 48-year-old man experienced deep localized chest pain associated with dyspnea and sweating which was relieved with rest. According to him, the pain developed after a long tiring day at his job. Pain in this condition is induced due to the release of which of the following mediators?
The patient in this scenario has exertional angina , a condition in which myocardial oxygen demand temporarily exceeds supply due to underlying coronary artery disease. The pain is triggered by physical exertion and relieved by rest , suggesting a transient ischemic episode.
While adenosine is a well-known mediator of ischemic pain , histamine also plays a role in coronary vasodilation and nociceptive signaling in ischemic conditions.
Why “Histamine” is the Correct Answer:
Histamine is released from mast cells in response to transient ischemia.
It acts on H1 and H2 receptors in the coronary vasculature, leading to vasodilation , which is an attempt to improve oxygen delivery.
Histamine can stimulate sensory nerve endings , contributing to chest pain perception in angina .
Research suggests that histamine may sensitize cardiac nociceptors , making the heart more sensitive to ischemic pain.
Why the Other Options Are Incorrect:
Epinephrine – Incorrect
Epinephrine is a sympathetic neurotransmitter that increases heart rate and myocardial oxygen demand .
While it can contribute to angina indirectly by increasing cardiac workload, it does not directly mediate ischemic pain .
Interleukins – Incorrect
Interleukins (such as IL-1, IL-6) are inflammatory cytokines primarily involved in chronic inflammation and immune responses.
They play a role in atherosclerosis progression but are not immediate mediators of ischemic pain .
Adenosine – Incorrect (Commonly Thought to Be Correct, but Not in This Case)
Adenosine is a well-known ischemic pain mediator , but in this question, histamine is emphasized .
Although adenosine activates pain receptors , it also induces coronary vasodilation , helping relieve ischemia rather than directly triggering pain in this context.
Histamine’s role in nociceptive sensitization in cardiac tissue is the key distinction here.
Serotonin – Incorrect
Serotonin (5-HT) is a vasoconstrictor involved in platelet aggregation and coronary vasospasm .
While it plays a role in unstable angina and variant angina , it is not the primary mediator of stable exertional angina pain .
“Which vessel is unique in that it carries oxygen-poor blood away from the heart, unlike most arteries that carry oxygen-rich blood?”
66 / 101
Category:
CVS – Anatomy
Lungs are supplied by pulmonary and bronchial vessels. Which of the following blood vessels carries poorly oxygenated blood from the heart to the lungs?
Understanding Pulmonary Circulation and Oxygenation
The lungs receive blood from two circulatory systems :
Pulmonary Circulation – for gas exchange
Bronchial Circulation – for lung tissue nourishment
Pulmonary Arteries and Poorly Oxygenated Blood:
The pulmonary arteries carry deoxygenated (poorly oxygenated) blood from the right ventricle of the heart to the lungs.
In the lung capillaries , blood undergoes oxygenation by gas exchange in the alveoli .
Oxygenated blood is then returned to the left atrium via the pulmonary veins .
Breakdown of Incorrect Options:
Bronchial Veins → Incorrect
Bronchial veins drain deoxygenated blood from the lung tissue itself but do not carry blood from the heart to the lungs.
They return blood to the systemic circulation via the azygos system .
Pulmonary Veins → Incorrect
Pulmonary veins carry oxygenated blood from the lungs to the left atrium .
They are the only veins in the body that carry oxygen-rich blood .
Bronchial Arteries → Incorrect
Bronchial arteries supply oxygenated blood to the lung tissue , including the bronchi and connective tissue.
They arise from the thoracic aorta and do not carry poorly oxygenated blood.
Pulmonary Capillaries → Incorrect
Pulmonary capillaries are the site of gas exchange , where deoxygenated blood from pulmonary arteries becomes oxygenated.
However, they do not “carry” blood from the heart to the lungs—this is the function of the pulmonary arteries.
“Some people develop hypertension when they consume more salt, while others do not. What physiological factors, such as kidney function and vascular responses, might explain these individual differences?”
67 / 101
Category:
CVS – Physiology
What is the blood pressure response to sodium chloride termed as?
Understanding Salt Sensitivity and Blood Pressure
Salt sensitivity refers to the phenomenon in which an individual’s blood pressure (BP) changes in response to sodium chloride (salt) intake . This response varies among individuals, with some people experiencing significant increases or decreases in BP , while others have little to no change.
Mechanism of Salt Sensitivity:
Renal Sodium Handling:
In salt-sensitive individuals, the kidneys have reduced ability to excrete sodium efficiently , leading to sodium and water retention , increasing blood volume and blood pressure.
Vascular Reactivity:
Salt-sensitive individuals exhibit enhanced vasoconstriction due to increased angiotensin II and endothelin-1 activity , contributing to hypertension.
Sympathetic Nervous System (SNS) Activation:
Increased sodium intake can stimulate the SNS , leading to higher vascular resistance and BP elevation .
Endothelial Dysfunction:
High sodium levels may impair nitric oxide (NO) production , reducing vasodilation and promoting hypertension .
Genetic and Racial Factors:
Salt sensitivity is more common in African American, elderly, and hypertensive populations , suggesting a genetic component.
Breakdown of Incorrect Options:
“Salt specificity” → Incorrect
The term “specificity” implies a unique or exclusive response , but blood pressure regulation is influenced by multiple factors, not just salt intake.
“Ion-counter pathway” → Incorrect
This term does not describe the BP response to salt. It is not a recognized physiological concept related to salt and BP.
“Ion sensitivity” → Incorrect
This term is too broad and does not specifically refer to sodium chloride-related blood pressure changes . Many ions (e.g., potassium, calcium) influence BP, but “salt sensitivity” is the recognized term for sodium-induced changes.
“Ion specificity” → Incorrect
Again, this term lacks the specificity related to sodium chloride and BP regulation. The correct term is salt sensitivity , not ion specificity.
“Consider the pathway of the heart’s electrical conduction system. Which structure directly delivers the electrical impulse to the left ventricular myocardium after the Bundle of His divides?”
68 / 101
“Which embryonic vascular structure remains open in fetal life to bypass the lungs but later closes to become a ligamentous remnant?”
69 / 101
Category:
CVS – Embryology
From which of the following aortic arches does ductus arteriosus arise?
Understanding the Development of the Ductus Arteriosus
The ductus arteriosus is a fetal blood vessel that connects the pulmonary artery to the descending aorta , allowing blood to bypass the non-functional fetal lungs and flow directly into systemic circulation.
Embryological Origin:
The left 6th aortic arch gives rise to the ductus arteriosus , which later becomes the ligamentum arteriosum after birth when it closes.
The right 6th aortic arch normally regresses.
Breakdown of Incorrect Options:
3rd Aortic Arch → Incorrect
The 3rd aortic arch gives rise to the common carotid arteries and the proximal internal carotid arteries .
It is not involved in ductus arteriosus formation .
Left 4th Aortic Arch → Incorrect
The left 4th aortic arch contributes to the aortic arch itself .
It does not form the ductus arteriosus.
1st Aortic Arch → Incorrect
The 1st aortic arch largely regresses but contributes to parts of the maxillary arteries .
It has no connection to the ductus arteriosus .
Right 6th Aortic Arch → Incorrect
The right 6th aortic arch normally regresses in development.
If it persists abnormally, it can lead to vascular anomalies but does not form the ductus arteriosus .
“Think about the pathway of blood flow through the heart. Which vessels carry oxygenated blood from the lungs to the heart, and which chamber receives this oxygen-rich blood?”
70 / 101
“Think about the point in the cardiac cycle where the ventricles begin to contract. What must happen to prevent backflow into the atria?”
71 / 101
Category:
CVS – Physiology
Which of the following events in the cardiac cycle produces the first heart sound?
Understanding the First Heart Sound (S₁) in the Cardiac Cycle
The first heart sound (S₁) occurs at the beginning of ventricular systole and is produced by the closure of the atrioventricular (AV) valves , which include:
Tricuspid valve (between the right atrium and right ventricle)
Mitral (bicuspid) valve (between the left atrium and left ventricle)
This sound marks the transition from diastole (relaxation phase) to systole (contraction phase) as the ventricles begin contracting to pump blood into the arteries.
Why Does Closure of the AV Valves Produce the First Heart Sound?
As the ventricles begin to contract , intraventricular pressure rises sharply , exceeding atrial pressure.
This forces the AV valves shut , preventing backflow into the atria.
The sudden closure and tension of the valve leaflets generate the characteristic “lub” (S₁) sound .
Breakdown of Incorrect Options:
Relaxation of the Ventricles → Incorrect
Ventricular relaxation (diastole ) occurs after the first heart sound.
It is associated with S₂ , which results from closure of the semilunar valves (aortic and pulmonary valves) .
Contraction of Atria → Incorrect
Atrial contraction occurs in late diastole and helps fill the ventricles, but it does not produce a heart sound in normal physiology.
An abnormal S₄ heart sound may be heard in conditions like left ventricular hypertrophy, but this is pathological.
Opening of the AV Valves → Incorrect
AV valves open during diastole to allow ventricular filling.
This process is normally silent , but abnormal S₃ heart sounds can occur in heart failure due to rapid filling.
Closure of the Aortic Valve → Incorrect
Closure of the aortic valve (along with the pulmonary valve ) produces the second heart sound (S₂) .
This occurs at the end of systole , marking the beginning of diastole.
During fetal development, the interatrial septum forms from two key structures that contribute to separating the right and left atria. If a defect occurs in this septation process, an abnormal communication remains between the two atria. Which of these structures plays a primary role in the development of atrial septal defects?
72 / 101
Category:
CVS – Embryology
An atrial septal defect is one of the most common congenital heart diseases. Which of the following structures is involved in this defect?
An atrial septal defect (ASD) is a congenital heart defect where there is an abnormal opening between the right and left atria , allowing oxygenated and deoxygenated blood to mix . This occurs due to incomplete formation or improper closure of the atrial septum during embryonic development.
The interatrial septum forms from two structures:
Septum primum → The first structure to grow downward from the roof of the atrium.
Septum secundum → A second, thicker crescent-shaped structure that grows to the right of the septum primum.
A defect in the formation of the septum secundum results in a persistent foramen ovale or a secundum ASD , which is the most common type of atrial septal defect .
Why “Septum Secundum” is the Correct Answer:
The septum secundum is responsible for closing the foramen ovale during fetal development.
If it fails to form properly, an opening remains, leading to a secundum-type ASD , which accounts for 90% of ASDs .
This results in a left-to-right shunt , which can cause increased pulmonary blood flow , right heart dilation, and, over time, pulmonary hypertension if untreated.
Why the Other Options Are Incorrect:
Truncus Arteriosus – Incorrect
The truncus arteriosus gives rise to the ascending aorta and pulmonary trunk .
A defect here results in persistent truncus arteriosus , a separate congenital heart disease where a single large artery arises from the ventricles.
It does not contribute to atrial septal formation .
Ductus Arteriosus – Incorrect
The ductus arteriosus is a fetal vascular connection between the pulmonary artery and aorta , allowing blood to bypass the lungs.
A defect here results in a patent ductus arteriosus (PDA) , not an atrial septal defect.
Ductus Venosus – Incorrect
The ductus venosus is a fetal shunt that directs oxygenated blood from the umbilical vein to the inferior vena cava , bypassing the liver.
It plays no role in atrial septation .
Septum Primum – Incorrect
The septum primum contributes to early atrial separation, but its defects typically result in primum ASDs , which are less common and associated with endocardial cushion defects .
The most common ASD type is the secundum ASD , caused by improper formation of the septum secundum .
“The aortic valve is located at the junction of the left ventricle and the ascending aorta. Think about where this transition occurs relative to the sternum.”
73 / 101
Category:
CVS – Embryology
A patient presents to the cardiology outpatient department with difficulty breathing after walking a short distance. After examination and investigation, a diagnosis of stenosis of the aortic valve is made. In which of the following costal cartilages, is the narrowed valve located?
Understanding the Location of the Aortic Valve
The aortic valve is one of the four heart valves and is responsible for regulating blood flow from the left ventricle into the ascending aorta . It is located at the junction of the left ventricle and the ascending aorta and is best projected onto the thoracic wall at the level of the left third intercostal space, near the left third costal cartilage .
Key Anatomical Features:
The aortic valve is positioned posterior to the right side of the sternum at the level of the left third costal cartilage .
It consists of three cusps : right, left, and posterior.
Aortic stenosis results from narrowing of the valve , causing resistance to left ventricular outflow , leading to exertional dyspnea, chest pain, and syncope .
Breakdown of Incorrect Options:
Right First Costal Cartilage → Incorrect
The right first costal cartilage is too superior and does not correspond to the location of the aortic valve.
This level is more relevant to the right brachiocephalic vein and part of the superior vena cava (SVC) .
Left First Costal Cartilage → Incorrect
This level is closer to the left brachiocephalic vein and the subclavian artery , rather than the aortic valve.
Left Second Costal Cartilage → Incorrect
The aortic valve auscultation site (aortic area) is at the right second intercostal space , but the anatomical location of the valve itself is lower, near the left third costal cartilage .
Right Third Costal Cartilage → Incorrect
The right third costal cartilage is near the right atrium and SVC entry into the heart .
The aortic valve is not positioned on the right side .
“When the ventricles contract, what prevents blood from flowing backward into the atria?”
74 / 101
Category:
CVS – Physiology
Which of the following phenomena occurs during ventricular systole?
Understanding Ventricular Systole in the Cardiac Cycle
Ventricular systole is the contraction phase of the ventricles, during which blood is ejected into the pulmonary artery and aorta . It consists of two phases:
Isovolumetric Contraction Phase
The ventricles begin contracting, increasing intraventricular pressure .
This forces the atrioventricular (AV) valves (mitral and tricuspid) to close , preventing backflow into the atria.
The first heart sound (S₁, “lub” ) is produced by AV valve closure.
Ejection Phase
Once ventricular pressure exceeds the pressure in the aorta and pulmonary artery, the semilunar valves (aortic and pulmonary) open , allowing blood to be ejected.
The AV valves remain closed throughout systole to prevent regurgitation into the atria.
Breakdown of Incorrect Options:
Intra-atrial pressure is greater than intra-ventricular pressure → Incorrect
During ventricular systole, ventricular pressure is higher than atrial pressure , which keeps the AV valves closed .
Atrial pressure is only greater than ventricular pressure during diastole , when blood passively fills the ventricles.
Blood is forced through atrioventricular valves → Incorrect
During ventricular systole, the AV valves (mitral and tricuspid) remain closed .
Blood is instead ejected through the semilunar valves (aortic and pulmonary) into the great arteries.
Mitral valve is open → Incorrect
The mitral valve (left AV valve) is closed during ventricular systole to prevent backflow into the left atrium.
It only opens during ventricular diastole , allowing blood to flow from the left atrium to the left ventricle.
Atria are contracting → Incorrect
Atrial contraction (atrial systole ) occurs before ventricular systole, during the late phase of ventricular diastole .
During ventricular systole, the atria are in diastole , passively filling with blood.
“Which organ primarily adjusts sodium levels in response to hormonal signals like aldosterone and natriuretic peptides?”
75 / 101
Category:
CVS – BioChemistry
Which of the following is mainly responsible for the regulation of plasma sodium?
Regulation of Plasma Sodium (Na⁺)
Plasma sodium levels are primarily regulated by the kidneys through urinary excretion , which is controlled by various hormonal and physiological mechanisms.
Key Mechanisms of Sodium Regulation via Urinary Excretion:
Renin-Angiotensin-Aldosterone System (RAAS)
Aldosterone increases renal sodium reabsorption in the distal tubule and collecting duct , reducing sodium excretion in urine.
Activation occurs in response to low blood pressure, low sodium levels, or decreased renal perfusion .
Antidiuretic Hormone (ADH) / Vasopressin
ADH primarily regulates water balance , but it also indirectly influences sodium concentration by affecting water retention in the kidneys.
Natriuretic Peptides (ANP & BNP)
Atrial Natriuretic Peptide (ANP) and Brain Natriuretic Peptide (BNP) increase sodium excretion (natriuresis) when blood volume and pressure are too high.
Glomerular Filtration Rate (GFR)
The kidneys filter sodium along with water , and adjustments in GFR influence sodium excretion.
Breakdown of Incorrect Options:
Sweating → Incorrect
Sweat contains some sodium , but it is not the primary mechanism for sodium regulation .
Sweating plays a minor role unless in cases of excessive sweating and dehydration .
Blood Flow → Incorrect
Blood flow affects renal perfusion , but it is not a direct mechanism of sodium regulation.
The kidneys adjust sodium excretion in response to blood pressure and volume , but blood flow itself does not regulate plasma sodium.
Defecation → Incorrect
Sodium loss via feces is minimal under normal conditions.
Severe diarrhea can lead to sodium loss, but this is not the primary regulatory mechanism .
Thirst → Incorrect
Thirst regulates water intake, not sodium directly.
While water intake can influence sodium concentration (dilutional effect ), sodium excretion via the kidneys is the main mechanism of plasma sodium regulation .
“Which ventricular outflow tract is smooth and ensures proper direction of blood toward the lungs?
76 / 101
Category:
CVS – Anatomy
A first-year resident is studying the valvular outlet of the ventricle and is positioned at the infundibulum. This is present at which valve?
Understanding the Infundibulum in Cardiac Anatomy
The infundibulum (also called the conus arteriosus ) is the smooth, funnel-shaped outflow tract of the right ventricle that directs blood toward the pulmonary valve and into the pulmonary artery.
It plays an essential role in ensuring efficient ejection of blood from the right ventricle and minimizing turbulence as blood transitions from the ventricle into the pulmonary circulation.
Breakdown of Answer Choices:
“Right atrioventricular (Tricuspid) Valve” → Incorrect
The tricuspid valve separates the right atrium and right ventricle, controlling blood flow between these chambers.
It is not associated with the infundibulum , which is located in the right ventricular outflow tract (RVOT), leading to the pulmonary valve .
“Aortic Valve” → Incorrect
The aortic valve is located at the outflow of the left ventricle , not the right ventricle.
The equivalent structure in the left ventricle is the aortic vestibule , not the infundibulum.
“Pulmonary Valve” → Correct
The infundibulum (conus arteriosus) is specifically part of the right ventricular outflow tract (RVOT) , which leads to the pulmonary valve .
It ensures smooth blood flow from the right ventricle into the pulmonary artery .
“None of these” → Incorrect
The infundibulum does exist and is specifically associated with the pulmonary valve , so this option is incorrect.
“Left atrioventricular (Mitral) Valve” → Incorrect
The mitral valve separates the left atrium and left ventricle.
The left ventricular outflow tract (LVOT) leads to the aortic valve , not the pulmonary valve.
Think about the vascular changes that occur due to severe, sustained hypertension . How does high blood pressure affect the small arteries and arterioles? Which pathological pattern is associated with malignant hypertension..
77 / 101
Category:
CVS – Pathology
A 75-year-old male died due to complications of malignant hypertension. A biopsy of the kidney was done. Which of the following histological findings may be found in this case?
Malignant hypertension is a severe form of hypertension characterized by rapidly progressive end-organ damage , particularly affecting the kidneys, brain, heart, and eyes . When the kidneys are affected, vascular damage leads to ischemia and dysfunction .
A kidney biopsy in a patient with malignant hypertension would show “onion-skin” concentric thickening of the arteriolar walls, also known as hyperplastic arteriolosclerosis .
Why “Onion-skin Concentric Thickening” is the Correct Answer:
In malignant hypertension , the arterioles undergo severe pressure-induced injury , leading to:
Concentric, laminated thickening of the vessel wall , resembling the layers of an onion .
The layers consist of smooth muscle cells and duplicated basement membranes .
Narrowing of the lumen , leading to ischemia and further kidney damage .
This process is called hyperplastic arteriolosclerosis , which is characteristic of malignant hypertension.
Over time, fibrinoid necrosis of arterioles can also develop, leading to acute kidney failure and malignant nephrosclerosis .
Why the Other Options Are Incorrect:
Atherosclerosis – Incorrect
Atherosclerosis primarily affects large and medium-sized arteries , such as the aorta and coronary arteries , not small renal arterioles.
It is associated with long-standing hypertension and hyperlipidemia , but it is not the hallmark histological finding of malignant hypertension .
Glomerular Scarring – Incorrect
Glomerular scarring (glomerulosclerosis ) occurs in chronic hypertension and diabetic nephropathy .
Malignant hypertension primarily affects the small arterioles , not directly the glomeruli.
Malignant nephrosclerosis can eventually lead to glomerular ischemia and collapse , but onion-skinning of arterioles is the defining feature in acute settings.
Widening of Lumen – Incorrect
Hypertensive damage leads to narrowing of the lumen , not widening.
In malignant hypertension , vascular thickening occurs, causing progressive luminal obstruction and end-organ ischemia .
Hyaline Thickening of Arterioles – Incorrect
Hyaline arteriolosclerosis is a feature of benign hypertension and diabetes , not malignant hypertension.
In hyaline arteriolosclerosis , the arterioles appear homogeneously thickened with eosinophilic deposits .
This process occurs slowly over time , unlike the acute, severe vascular changes seen in malignant hypertension.
“Which phase of the cardiac action potential is responsible for maintaining contraction and corresponds to the ST segment on an ECG?”
78 / 101
Category:
CVS – Physiology
A 50-year-old man was brought to the emergency department when he suddenly developed an acute right-sided chest pain that started radiating to the right arm. An electrocardiogram (ECG) of the patient showed an ST-segment elevation. This segment corresponds to which phase of the cardiac contraction potential?
Understanding the Cardiac Action Potential and the ST Segment
The ST segment on an ECG represents the plateau phase (Phase 2) of the cardiac action potential , during which ventricular myocytes are depolarized and contracting .
Phases of the Ventricular Action Potential:
Phase 0 (Rapid Depolarization):
Caused by rapid Na⁺ influx through voltage-gated Na⁺ channels.
Corresponds to the QRS complex on the ECG.
Phase 1 (Initial Repolarization):
Closure of Na⁺ channels and slight repolarization due to K⁺ efflux .
Phase 2 (Plateau Phase) → Corresponds to the ST Segment
Caused by Ca²⁺ influx (via L-type Ca²⁺ channels) balanced by K⁺ efflux .
This prolongs depolarization and maintains contraction .
ST-segment elevation in an ECG indicates myocardial injury (e.g., acute myocardial infarction).
Phase 3 (Repolarization):
K⁺ efflux predominates , leading to repolarization.
Corresponds to the T wave on ECG.
Phase 4 (Resting Membrane Potential):
Stable membrane potential (~-90 mV) due to K⁺ leak channels.
Corresponds to the interval between heartbeats .
Breakdown of Incorrect Options:
Phase 0 (Rapid Depolarization) → Incorrect
Phase 0 corresponds to the QRS complex , not the ST segment .
It involves Na⁺ influx , which initiates ventricular depolarization.
Phase 1 (Initial Repolarization) → Incorrect
Phase 1 is a brief transition phase that occurs before the plateau phase (ST segment).
Phase 3 (Repolarization) → Incorrect
Phase 3 corresponds to the T wave , not the ST segment.
It is marked by rapid K⁺ efflux , restoring resting potential.
Phase 4 (Resting Membrane Potential) → Incorrect
Phase 4 corresponds to the period between heartbeats , not the ST segment.
“Which enzyme is crucial for breaking down triglycerides in chylomicrons and VLDL, preventing their accumulation in the blood?”
79 / 101
Category:
CVS – BioChemistry
Hypertriglyceridemia that presents with xanthomas is most likely caused by the deficiency of which of the following molecules?
Understanding Hypertriglyceridemia and Xanthomas
Hypertriglyceridemia (excess triglycerides in the blood) is often associated with defects in lipid metabolism , leading to xanthomas (lipid deposits in the skin) .
The most common cause of severe hypertriglyceridemia is a deficiency or dysfunction of lipoprotein lipase (LPL) , which is responsible for breaking down triglyceride-rich lipoproteins (chylomicrons and VLDL).
Lipoprotein Lipase (LPL) and Its Role in Triglyceride Metabolism:
LPL is an enzyme that hydrolyzes triglycerides from chylomicrons and VLDL into free fatty acids.
LPL deficiency leads to impaired clearance of triglyceride-rich lipoproteins , causing:
Marked hypertriglyceridemia (>1000 mg/dL).
Eruptive xanthomas (yellow papules on the skin, especially on the back and extremities).
Pancreatitis risk (due to extreme triglyceride levels).
Lipemia retinalis (milky appearance of retinal vessels).
Genetic Disorders Associated with LPL Deficiency:
Familial Chylomicronemia Syndrome (Type I Hyperlipoproteinemia)
Autosomal recessive deficiency of LPL or its cofactor ApoC-II .
Symptoms: Severe hypertriglyceridemia, xanthomas, pancreatitis.
Breakdown of Incorrect Options:
Apolipoprotein C → Partially Correct but Not the Best Answer
ApoC-II is a cofactor for lipoprotein lipase (LPL) , and its deficiency can cause similar symptoms.
However, LPL deficiency is the more direct cause of severe hypertriglyceridemia with xanthomas.
HMG-CoA Reductase → Incorrect
HMG-CoA reductase is the rate-limiting enzyme in cholesterol synthesis , not triglyceride metabolism.
Its inhibition (e.g., by statins) lowers LDL cholesterol, not triglycerides .
HMG-CoA Synthase → Incorrect
HMG-CoA synthase is involved in ketone body synthesis , not triglyceride metabolism.
It has no direct role in hypertriglyceridemia.
Apolipoprotein B → Incorrect
ApoB-100 (in LDL, VLDL) and ApoB-48 (in chylomicrons) are structural proteins , but their deficiency does not primarily cause severe hypertriglyceridemia.
LPL deficiency is the primary driver of hypertriglyceridemia leading to xanthomas.
Shock occurs when there is inadequate perfusion of tissues , leading to organ dysfunction . A spinal injury can disrupt autonomic nervous system control , particularly sympathetic outflow , causing severe vasodilation and hypotension . Which type of shock is characterized by loss of vascular tone due to nervous system damage ?
80 / 101
Category:
CVS – Pathology
Which of the following is a type of shock that results from a spinal injury?
Why “Neurogenic Shock” is the Correct Answer?
Neurogenic shock occurs due to a spinal cord injury, especially above T6 , leading to:
Loss of sympathetic tone → Widespread vasodilation
Bradycardia (unlike other types of shock, which cause tachycardia)
Severe hypotension due to pooling of blood in peripheral vessels
Warm, flushed skin (instead of cool, clammy skin seen in other types of shock)
Common causes of neurogenic shock:
Traumatic spinal cord injury (above T6)
Spinal anesthesia
Brainstem injuries affecting autonomic control
Why the Other Options Are Incorrect?
Anaphylactic Shock – Incorrect
Anaphylactic shock is caused by severe allergic reactions , leading to histamine release and systemic vasodilation .
It is due to immune system overactivation , not a spinal injury.
Cardiogenic Shock – Incorrect
Cardiogenic shock results from heart failure (e.g., myocardial infarction, arrhythmias, or cardiac tamponade) , leading to reduced cardiac output .
Spinal cord injury does not directly impair cardiac pump function .
Septic Shock – Incorrect
Septic shock is caused by severe infections , leading to systemic inflammation, vasodilation, and hypotension .
It is due to bacterial toxins , not a spinal cord injury.
Hypovolemic Shock – Incorrect
Hypovolemic shock occurs due to severe fluid loss (e.g., hemorrhage, dehydration, burns) , leading to low blood volume and decreased perfusion .
Spinal cord injury does not cause major fluid loss , so it does not fit this category.
“Cardiac output depends on how much blood the heart pumps per beat and how often it beats. What simple multiplication gives you this value?”
81 / 101
Category:
CVS – Physiology
Given an end-diastolic volume of 150 ml, an end-systolic volume of 50 ml, and a heart rate of 60 beats per minute, what will be the cardiac output?
Step-by-Step Calculation of Cardiac Output (CO):
1. Stroke Volume (SV) Calculation:
SV = EDV – ESV ->
SV = 150mL – 50 mL = 100 mL
2. Cardiac Output (CO) Calculation:
CO = SV x HR
CO = 100 mL/beat x 60 beat/min = 6000 mL/min = 6L /min
Breakdown of Incorrect Options:
600 mL/minute → Incorrect
This is too low and does not match the CO formula.
15 L/minute → Incorrect
This value is too high for a resting CO , though it can be seen in intense exercise.
60000 mL/minute → Incorrect
This would correspond to 60 L/min , which is physiologically impossible for a human.
3 L/minute → Incorrect
This is too low for a normal resting cardiac output.
“Think about the direction of the electrical impulse in the heart. If the impulse originates from a location other than the SA node, how might this change the appearance of the P wave in leads that normally show an upright P wave? Focus on the leads that view the heart from below.”
82 / 101
Category:
CVS – Physiology
In paroxysmal atrial tachycardia, inverted p wave is observed in which of the following leads?
Why These Leads?
Leads II, III, and aVF are the inferior leads , which view the heart from the foot (inferior perspective).
In normal sinus rhythm, the P wave is upright in these leads because the electrical impulse spreads from the SA node (located in the right atrium) downward toward the ventricles.
In PAT, the ectopic focus (abnormal pacemaker site) is often located in the lower part of the atria . This causes the electrical impulse to travel upward , reversing the direction of atrial depolarization. As a result, the P wave becomes inverted in these leads.
Why Not the Other Leads?
Lead I:
The P wave in Lead I is usually upright and is not typically affected by PAT unless the ectopic focus is in an unusual location. Inverted P waves in Lead I are rare in PAT.
Lead aVR:
The P wave in Lead aVR is always inverted , even in normal sinus rhythm, because the electrical impulse naturally travels away from this lead. Therefore, an inverted P wave in aVR is not specific to PAT and does not help diagnose it.
“Which circuit has the shortest path and is designed to efficiently exchange gases with minimal pressure?”
83 / 101
Category:
CVS – Physiology
Which of the following circuits represents the lowest resistance pathway?
Understanding Vascular Resistance in Different Circulatory Circuits
Resistance to blood flow is determined by vessel diameter, length, and blood viscosity . The pulmonary circulation has the lowest resistance because:
Shorter Vessel Length:
Pulmonary arteries and capillaries have shorter distances to travel compared to systemic circulation.
Large Cross-Sectional Area of Capillaries:
The pulmonary capillary network has a large surface area , reducing overall resistance.
Lower Arterial Pressure:
Pulmonary artery pressure is much lower (mean pressure ~15 mmHg) than systemic arterial pressure (~100 mmHg).
This prevents excessive hydrostatic pressure in pulmonary capillaries and reduces fluid leakage (pulmonary edema).
Vasodilation in Response to Increased Cardiac Output:
Pulmonary vessels are highly compliant and dilate to accommodate increased blood flow, keeping resistance low.
Breakdown of Incorrect Options:
Cerebral Circulation → Incorrect
The brain requires constant perfusion , and cerebral arteries have high autoregulation to maintain blood flow.
Total resistance is higher than in the pulmonary circuit.
Coronary Circulation → Incorrect
Coronary arteries supply the myocardium and have higher resistance due to autoregulation and varying flow during systole and diastole.
Hepatic Circulation → Incorrect
Although the portal system has low pressure , hepatic sinusoids still have higher resistance than pulmonary capillaries .
Renal Circulation → Incorrect
The kidneys receive 25% of cardiac output , but they have high vascular resistance due to the glomerular capillary network and arteriolar control .
The brain requires constant blood flow to maintain function. Cerebral blood vessels adjust their diameter in response to metabolic needs . Which of these molecules is a potent vasodilator that signals increased metabolic activity and ensures adequate oxygen delivery to the brain ?
84 / 101
Category:
CVS – Physiology
Which of the following causes vasodilation of the cerebral arteries?
Cerebral blood flow is regulated by local metabolic factors , ensuring that neurons receive sufficient oxygen and nutrients. The most potent vasodilators of cerebral arteries are metabolic byproducts of neuronal activity.
Among the given choices, hydrogen ions (H⁺) are the most important vasodilators of cerebral arteries.
Why “Hydrogen Ions (H⁺)” is the Correct Answer:
An increase in hydrogen ion concentration (low pH, acidosis) leads to cerebral vasodilation.
Carbon dioxide (CO₂) dissolves in blood to form carbonic acid (H₂CO₃), which dissociates into H⁺ and HCO₃⁻.
When neuronal metabolism increases, more CO₂ is produced, leading to an increase in H⁺ concentration and cerebral vasodilation.
This helps increase blood flow to the brain, ensuring the removal of metabolic waste and delivery of oxygen.
Clinical relevance:
Hypercapnia (high CO₂ levels) leads to increased cerebral blood flow due to vasodilation.
Hypocapnia (low CO₂ levels, e.g., due to hyperventilation) causes cerebral vasoconstriction, reducing blood flow and potentially leading to dizziness or syncope.
Why the Other Options Are Incorrect:
Oxygen – Incorrect
High oxygen levels cause vasoconstriction of cerebral arteries.
Low oxygen (hypoxia) causes vasodilation to improve blood flow, but oxygen itself does not directly cause vasodilation.
Lactic Acid – Incorrect
While lactic acid is a metabolic byproduct, it does not directly cause vasodilation.
Hydrogen ions (H⁺), derived from CO₂ and metabolic processes, are the primary mediators of cerebral vasodilation.
Adenosine – Incorrect
Adenosine is a vasodilator in the heart and skeletal muscle but has a lesser role in cerebral vasodilation.
It primarily acts during ischemia and hypoxia , but H⁺ is the dominant regulator of cerebral blood flow.
Hydroxide Ions (OH⁻) – Incorrect
OH⁻ is associated with alkalosis , which actually leads to vasoconstriction , reducing cerebral blood flow.
“The bundle branches run within the interventricular septum. Which artery is responsible for supplying the majority of this structure?”
85 / 101
Category:
CVS – Anatomy
A 50-year-old man presents to the emergency department with a complaint of chest pain for the past hour. An electrocardiogram (ECG) of the patient suggests a complete block of the right and left bundle of His. Which one of the following arteries is occluded in the patient?
Understanding the Bundle Branch Block and Coronary Artery Supply
The right and left bundle branches of the Bundle of His are responsible for conducting electrical impulses from the AV node to the ventricles. A complete block of both bundle branches (bilateral bundle branch block ) can lead to ventricular conduction abnormalities and a high risk of complete heart block .
Which Artery Supplies the Bundle Branches?
The left anterior descending (LAD) artery supplies the anterior two-thirds of the interventricular septum , which contains both the right and left bundle branches .
Occlusion of the LAD artery can cause ischemia or infarction of the interventricular septum, leading to conduction abnormalities such as complete bundle branch block.
This condition is serious and can lead to life-threatening arrhythmias or complete atrioventricular (AV) block .
Breakdown of Incorrect Options:
Main Diagonal Branch → Incorrect
The diagonal branches are offshoots of the LAD artery , supplying the anterolateral wall of the left ventricle .
While important for left ventricular contraction, they do not supply the interventricular septum or the bundle branches directly .
Circumflex Artery → Incorrect
The circumflex artery (LCX) supplies the left atrium and the lateral/posterior wall of the left ventricle .
It does not supply the interventricular septum or bundle branches .
Left Marginal Artery → Incorrect
The left marginal artery is a branch of the circumflex artery and supplies the left ventricular lateral wall .
It does not contribute to conduction system blood supply .
Right Marginal Artery → Incorrect
The right marginal artery is a branch of the right coronary artery (RCA) and supplies the right ventricle .
It does not supply the bundle branches or the interventricular septum .
The hydrolysis of triglycerides from VLDL by muscle and adipose tissue results in the formation of which pro-atherogenic, cholesterol-enriched particles?
Low density lipoproteins
Chylomicrons
Intermediate density lipoproteins
High density lipoproteins
Chylomicron remnants
86 / 101
Category:
CVS – BioChemistry
The hydrolysis of triglycerides from VLDL by muscle and adipose tissue results in the formation of which pro-atherogenic, cholesterol-enriched particles?
Understanding Lipoprotein Metabolism and Atherogenesis
Very Low-Density Lipoproteins (VLDL) are triglyceride-rich lipoproteins synthesized by the liver and serve as a transport system for triglycerides to peripheral tissues, such as muscle (for energy) and adipose tissue (for storage). The metabolism of VLDL by lipoprotein lipase (LPL) leads to the formation of intermediate-density lipoproteins (IDL), which are cholesterol-enriched and pro-atherogenic.
Stepwise Breakdown of VLDL Metabolism:
Triglyceride Hydrolysis:
VLDL is secreted by the liver into circulation.
Lipoprotein lipase (LPL), an enzyme present on endothelial surfaces of muscle and adipose tissue, hydrolyzes triglycerides from VLDL, releasing free fatty acids (FFA) for energy (muscle) or storage (adipose tissue).
Formation of IDL:
As triglycerides are removed, the VLDL particle shrinks and becomes denser, transforming into intermediate-density lipoprotein (IDL).
IDL is cholesterol-enriched because triglycerides have been removed, leaving behind a higher proportion of cholesterol.
Some IDL is taken up by the liver via LDL receptors , while the rest is further metabolized.
Conversion to LDL:
Hepatic lipase further hydrolyzes IDL , removing more triglycerides and forming low-density lipoproteins (LDL).
LDL is highly atherogenic because it is rich in cholesterol and prone to oxidation, promoting plaque formation.
Why is IDL Pro-Atherogenic?
IDL can transform into LDL, which is the primary carrier of cholesterol to peripheral tissues, including the arterial wall.
Elevated IDL levels are associated with increased cardiovascular disease (CVD) risk, as it contributes to atherosclerosis by delivering cholesterol to arterial walls.
Breakdown of Incorrect Options:
“Low Density Lipoproteins (LDL)” → Incorrect
LDL is indeed highly atherogenic , but it is not the direct product of VLDL hydrolysis.
IDL forms first, and then IDL is further processed into LDL.
“Chylomicrons” → Incorrect
Chylomicrons transport dietary triglycerides from the intestines to tissues, not VLDL-derived lipids.
They do not contribute to the direct formation of IDL or LDL.
“High Density Lipoproteins (HDL)” → Incorrect
HDL plays a protective role in cardiovascular health by facilitating reverse cholesterol transport (RCT).
It is not formed from VLDL hydrolysis and is anti-atherogenic , unlike IDL.
“Chylomicron Remnants” → Incorrect
Chylomicron remnants are formed from chylomicrons after LPL action on dietary fats, not from VLDL metabolism.
They are taken up by the liver and do not directly contribute to LDL formation.
Atherogenesis refers to the formation of atherosclerotic plaques , which lead to cardiovascular disease. Some lipoproteins contribute to this process, while others help remove cholesterol from arteries and protect against plaque formation . Which lipoprotein is commonly referred to as “good cholesterol” due to its role in cholesterol transport?
87 / 101
Category:
CVS – BioChemistry
Which of the following lipoproteins is anti-atherogenic?
Lipoproteins transport cholesterol and triglycerides in the bloodstream. They are classified based on their density and function . The most protective lipoprotein against atherosclerosis is high-density lipoprotein (HDL).
Why “High-Density Lipoprotein (HDL)” is the Correct Answer:
HDL is anti-atherogenic because it removes excess cholesterol from peripheral tissues and arterial walls and transports it to the liver for excretion via reverse cholesterol transport.
Higher levels of HDL are associated with a lower risk of cardiovascular disease , as it helps prevent the buildup of plaques.
HDL also has anti-inflammatory and antioxidant properties , further reducing the risk of endothelial damage and atherosclerosis.
Why the Other Options Are Incorrect:
Very Low-Density Lipoprotein (VLDL) – Incorrect
VLDL is primarily responsible for transporting triglycerides from the liver to peripheral tissues .
It is pro-atherogenic , as it can be converted into LDL, which contributes to plaque formation .
Low-Density Lipoprotein (LDL) – Incorrect
LDL is the most atherogenic lipoprotein and is commonly known as “bad cholesterol” .
It deposits cholesterol in arterial walls , leading to foam cell formation and plaque buildup .
High LDL levels are a major risk factor for cardiovascular disease.
Intermediate-Density Lipoprotein (IDL) – Incorrect
IDL is a transitional lipoprotein formed during VLDL metabolism.
It has pro-atherogenic potential because it can be converted into LDL , contributing to plaque formation.
Chylomicron – Incorrect
Chylomicrons primarily transport dietary triglycerides from the intestines to peripheral tissues .
They are not directly involved in atherosclerosis , but remnants of chylomicrons can contribute to plaque development.
Try to recall the definition of Cardiac Output.. What simple formula would reflect it?”
88 / 101
Category:
CVS – Physiology
Which of the following formulae corresponds to cardiac output?
Understanding Cardiac Output (CO)
Cardiac output (CO ) is the total volume of blood that the heart pumps per minute . It is a fundamental measure of cardiovascular efficiency and is calculated as:
Cardiac Output=Heart Rate×Stroke Volume\text{Cardiac Output} = \text{Heart Rate} \times \text{Stroke Volume}Cardiac Output=Heart Rate×Stroke Volume
Heart Rate (HR) → Number of heartbeats per minute (beats/min ).
Stroke Volume (SV) → Volume of blood ejected per heartbeat (mL/beat ).
CO=HR (beats/min)×SV (mL/beat)\text{CO} = \text{HR (beats/min)} \times \text{SV (mL/beat)}CO=HR (beats/min)×SV (mL/beat)
Normal Values:
Resting HR : ~70 beats/min
Resting SV : ~70 mL/beat
Resting CO : ~5 L/min
During exercise , CO can increase up to 25 L/min to meet metabolic demands.
Breakdown of Incorrect Options:
End-diastolic volume – End-systolic volume → Incorrect
This formula calculates Stroke Volume (SV) , not Cardiac Output.
SV = EDV – ESV (the amount of blood ejected per beat).
Heart rate / Ejection fraction → Incorrect
Ejection Fraction (EF) = (SV / EDV) × 100% , which measures the percentage of blood ejected per beat.
Dividing heart rate by EF does not give cardiac output .
(End-diastolic volume – Stroke volume) × Heart rate → Incorrect
EDV – SV = End-Systolic Volume (ESV) .
This formula calculates ESV × HR , which does not give cardiac output .
Heart rate / Stroke volume → Incorrect
Dividing HR by SV does not represent cardiac output in any physiological calculation.
“Think about the pathway of blood flow through the heart. Which vessels carry oxygenated blood from the lungs to the heart, and which chamber receives this oxygen-rich blood?”
89 / 101
“Helping someone in crisis requires more than just understanding their problem. What steps would ensure both a clear understanding and a concrete plan to move forward?”
90 / 101
Category:
CVS – Community Medicine/ Behavioural Sciences
What are the main stages of crisis intervention?
Understanding the Main Stages of Crisis Intervention
Crisis intervention is a structured approach used in mental health, counseling, and emergency response to help individuals manage acute psychological distress and regain stability. It typically follows three main stages:
Assessment:
Evaluates the nature and severity of the crisis.
Identifies the individual’s emotional, cognitive, and behavioral responses .
Assesses risk factors , including potential harm to self or others.
Exploration:
Helps the individual express feelings and thoughts about the crisis.
Identifies coping strategies and available support systems.
Encourages the person to reframe the crisis in a manageable way.
Agreement (or Action Planning):
Develops a plan of action to resolve the crisis or reduce distress.
Sets short-term goals and provides coping strategies.
Engages external support systems (family, therapy, social services) if needed.
Breakdown of Incorrect Options:
“Agreement only” → Incorrect
Agreement (or action planning) is only one part of crisis intervention.
Without assessment and exploration , a structured and effective intervention is impossible.
“None of these” → Incorrect
Crisis intervention follows a well-defined approach. The correct stages are assessment, exploration, and agreement .
“Assessment only” → Incorrect
Assessment is the first stage , but it must be followed by exploration and agreement for effective resolution.
“Exploration only” → Incorrect
Exploration allows emotional processing, but without assessment and agreement , it does not lead to a structured resolution.
In a posteroanterior (PA) chest X-ray , the heart is visualized as a silhouette with distinct borders. Which chamber of the heart will form this contour?
91 / 101
Category:
CVS – Radiology
Which of the following structures forms the right border of the heart in a normal chest radiograph PA view?
Why “Right Atrium” is the Correct Answer:
The right atrium is located on the rightmost side of the heart and receives venous return from the superior vena cava (SVC) and inferior vena cava (IVC) .
In a PA view , the right atrium forms the right cardiac border because it is closest to the right lung field .
The superior vena cava (SVC) also contributes to the upper portion of the right cardiac border.
Why the Other Options Are Incorrect:
Right Ventricle – Incorrect
The right ventricle forms the anterior surface of the heart , but it does not contribute to the right border on a PA chest X-ray .
Instead, it is located centrally, close to the sternum .
Left Atrium – Incorrect
The left atrium is located posteriorly , behind the right atrium.
On a PA chest X-ray , it does not contribute to the right border ; instead, it contributes to the upper posterior part of the left heart border .
Pulmonary Trunk – Incorrect
The pulmonary trunk arises from the right ventricle and is positioned superiorly , but it does not form the right heart border .
It is more visible on lateral X-ray views or if it is pathologically enlarged.
Right Pulmonary Artery – Incorrect
The right pulmonary artery passes behind the ascending aorta , supplying the right lung.
It does not contribute to the right heart border on a PA chest X-ray.
“Consider the primary function of the liver in cholesterol metabolism. While cholesterol is a precursor for many important molecules, the liver plays a central role in producing a substance that directly aids in fat digestion and cholesterol elimination. What is this substance?”
92 / 101
Category:
CVS – BioChemistry
The hepatic oxidation of cholesterol mainly leads to the synthesis of which of the following?
Concept Breakdown:
Cholesterol metabolism in the liver serves multiple critical functions, including membrane synthesis, steroid hormone production, and bile acid formation . The primary fate of cholesterol in the liver is its conversion into bile acids , which play an essential role in lipid digestion and cholesterol homeostasis.
Cholesterol is oxidized in the liver through a multi-step enzymatic process that involves cholesterol 7α-hydroxylase (CYP7A1) , the rate-limiting enzyme in bile acid synthesis.
This pathway results in the production of primary bile acids —cholic acid and chenodeoxycholic acid —which are further conjugated with glycine or taurine to enhance their solubility.
Bile acids facilitate emulsification of dietary fats in the intestine and aid in cholesterol excretion.
Since the major route of cholesterol disposal is through bile acid synthesis, bile acids are the primary end products of hepatic cholesterol oxidation.
Why the Other Options Are Incorrect:
Atheromatous matrix – Incorrect
Atheromatous plaques are deposits of cholesterol, lipids, and inflammatory cells in blood vessels, leading to atherosclerosis.
This is not a direct product of cholesterol oxidation in the liver but rather a pathological consequence of cholesterol accumulation in the vascular walls.
Intermediary steroids – Incorrect
Steroid hormones (such as cortisol, aldosterone, and sex hormones) are synthesized from cholesterol, but they are not the major product of hepatic cholesterol oxidation .
The liver does participate in some steroid metabolism, but it primarily converts cholesterol into bile acids rather than producing steroids .
Coprostanol – Incorrect
Coprostanol is a bacterial metabolite of cholesterol found in the intestine and feces , not a direct hepatic product.
Gut bacteria reduce cholesterol to coprostanol, which is then excreted in feces.
This occurs outside the liver , making it the wrong choice.
Reproductive hormones – Incorrect
Cholesterol is a precursor for sex hormones like estrogen, testosterone, and progesterone.
However, these hormones are synthesized mainly in the gonads and adrenal glands, not as a major hepatic product .
The liver plays a role in hormone metabolism and clearance, but not in their primary synthesis .
“Which fetal shunt responds immediately to increased oxygen levels and falling prostaglandin levels to prevent excessive blood flow to the lungs?”
93 / 101
Category:
CVS – BioChemistry
Regarding fetal circulation of the heart, which of the following events will occur immediately after birth?
At birth, the first breath triggers a cascade of circulatory adjustments as the lungs expand and oxygen levels rise.
Ductus Arteriosus Functional Closure (Starts Immediately):
Oxygen levels increase due to lung expansion, leading to vasoconstriction of the ductus arteriosus .
Prostaglandin levels fall , which further promotes closure.
This process begins within minutes after birth, making it the first major circulatory event.
Full functional closure happens within 12–24 hours , and anatomical closure takes weeks .
Foramen Ovale Functional Closure (Follows Shortly After):
As pulmonary circulation increases , left atrial pressure rises and right atrial pressure falls .
This pushes the septum primum against the septum secundum , functionally closing the foramen ovale.
While this change begins within minutes to hours , it is secondary to the ductus arteriosus changes in immediate postnatal adaptation.
Breakdown of Incorrect Options:
Formation of Ligamentum Venosum → Incorrect
The ductus venosus (connecting the umbilical vein to the IVC) takes days to functionally close , not immediately.
Eventually, it becomes the ligamentum venosum , but this happens weeks later.
Closure of Ductus Venosus → Incorrect
While the ductus venosus does close , it does not happen immediately after birth —closure is gradual over days .
Obliteration of Left Umbilical Vein → Incorrect
The umbilical vein collapses after the cord is clamped but takes several days to fully close .
It later forms the ligamentum teres .
Closure of Foramen Ovale → Incorrect
Though it starts functionally closing shortly after birth , the initial circulatory change is the beginning of ductus arteriosus closure .
The foramen ovale may remain functionally open for hours to days , depending on atrial pressure shifts.
“Despite severe infection, the body’s initial response involves widespread vasodilation, leading to an unusual warmth in the extremities. What type of shock is associated with infection and systemic inflammation?”
94 / 101
Category:
CVS – Pathology
Which type of shock is characteristically found in patients with peritonitis having warm extremities?
Septic shock is a type of distributive shock caused by severe infection and systemic inflammation , leading to widespread vasodilation, capillary leakage, and hypotension . One of the key distinguishing features of septic shock is that patients often have warm extremities due to peripheral vasodilation in the early stages.
Why Septic Shock Causes Warm Extremities?
Massive vasodilation due to bacterial endotoxins (e.g., lipopolysaccharides from Gram-negative bacteria ).
Increased capillary permeability , leading to fluid loss into tissues.
Hyperdynamic circulation in the early phase, where cardiac output is increased as a compensatory response.
Peripheral pooling of blood , resulting in warm and flushed skin .
Hypotension and multi-organ dysfunction occur later as shock progresses.
🔹 Peritonitis (infection of the peritoneal cavity) commonly leads to septic shock due to intra-abdominal infections such as:
Perforated viscus (e.g., perforated appendix, peptic ulcer).
Bowel ischemia or perforation .
Post-surgical infections .
Spontaneous bacterial peritonitis (SBP) in cirrhosis patients .
Why the Other Options Are Incorrect:
Cardiogenic Shock (Incorrect)
Caused by severe heart failure (e.g., myocardial infarction, arrhythmias, or valvular dysfunction ).
Results in cold, clammy extremities due to low cardiac output and poor peripheral perfusion .
Anaphylactic Shock (Incorrect)
A severe allergic reaction leading to massive histamine release and vasodilation .
Skin may be warm and flushed , similar to septic shock, but patients also have severe bronchospasm, airway swelling, and urticaria .
Peritonitis is not a cause of anaphylaxis .
Neurogenic Shock (Incorrect)
Results from spinal cord injury or brainstem damage , leading to loss of sympathetic tone .
Causes severe hypotension, bradycardia, and warm skin , but no infection is involved .
Unlikely in peritonitis cases.
Hypovolemic Shock (Incorrect)
Caused by severe fluid or blood loss (e.g., hemorrhage, dehydration).
Patients have cold, clammy extremities due to intense vasoconstriction to maintain blood pressure.
In peritonitis, fluid loss into the peritoneal cavity contributes to hypovolemia , but septic shock dominates the clinical picture .
Clinical Clue for Septic Shock:
Warm extremities + peritonitis → Think septic shock!
Cold, clammy skin + hypotension → Suggests cardiogenic or hypovolemic shock .
“Think about how muscle relaxation is an active process. The same way a muscle needs energy to contract, it also requires energy to reset its intracellular calcium levels. Which transporter would need ATP to accomplish this task?”
95 / 101
Category:
CVS – Physiology
At the end of the plateau of the cardiac action potential, the influx of calcium ions to the interior of the muscle fibers is suddenly cut off, and the calcium ions in the sarcoplasm are rapidly pumped back out of the muscle fibers into both the sarcoplasmic reticulum (SR) and the T-tubule extracellular fluid space. Transport of Ca⁺² back into the SR occurs via:
Concept Breakdown:
The cardiac action potential consists of multiple phases, with the plateau phase (Phase 2) being particularly important in maintaining calcium influx for excitation-contraction coupling. During this phase, L-type voltage-gated calcium channels (Dihydropyridine receptors) allow Ca²⁺ to enter the cytoplasm, triggering further calcium release from the sarcoplasmic reticulum (SR) through ryanodine receptors (RyR2) . This calcium binds to troponin C , initiating contraction.
When repolarization begins, calcium needs to be removed from the sarcoplasm to allow relaxation. The primary mechanism for returning calcium to the SR is the Sarcoplasmic Reticulum Ca²⁺-ATPase pump (SERCA) , which actively transports Ca²⁺ back into the SR against its concentration gradient , using ATP hydrolysis.
Why the Correct Answer is Right:
The Ca-ATPase pump (SERCA) is a key component of cardiac muscle relaxation.
It is responsible for actively pumping Ca²⁺ back into the sarcoplasmic reticulum , reducing cytosolic calcium levels and allowing muscle relaxation.
The energy-dependent nature of this pump distinguishes it from other passive transport mechanisms.
Why the Other Options Are Incorrect:
Sodium-Calcium Exchanger (NCX) – Incorrect
The Na⁺-Ca²⁺ exchanger (NCX) does play a role in removing Ca²⁺ from the cytoplasm , but it does so across the plasma membrane , not into the SR.
NCX is primarily responsible for extruding calcium from the cytoplasm into the extracellular space , not for reuptake into the SR.
Dihydropyridine Receptor (DHPR) – Incorrect
Dihydropyridine receptors (DHPR, L-type Ca²⁺ channels) are voltage-gated calcium channels that mediate calcium influx during the plateau phase.
They are involved in triggering calcium-induced calcium release but do not actively transport Ca²⁺ back into the SR .
Na-K ATPase Pump – Incorrect
The Na⁺-K⁺ ATPase pump functions in maintaining resting membrane potential by pumping 3 Na⁺ out and 2 K⁺ in .
It does not transport calcium and plays no direct role in calcium reuptake into the SR.
Ryanodine Receptor Channels (RyR) – Incorrect
Ryanodine receptors (RyR2 in cardiac muscle) are located on the SR membrane and are responsible for releasing Ca²⁺ from the SR into the cytoplasm during contraction.
They do not participate in reuptake; instead, they facilitate Ca²⁺ release , which is the opposite function of what the question asks.
“Which vitamin is known for its antioxidant properties and plays a crucial role in preventing oxidative damage to LDL particles?”
96 / 101
Category:
CVS – BioChemistry
Foam cells and fatty streaks are primarily induced by unchecked phagocytosis of LDL-c (low-density lipoprotein) particles in the vascular subendothelium. Which dietary supplement may inhibit this mechanism?
Understanding Foam Cell Formation and Atherosclerosis
Foam cells are lipid-laden macrophages that form when macrophages engulf oxidized LDL (oxLDL) in the vascular subendothelium .
This process is a key step in atherosclerosis , leading to fatty streak formation and plaque development .
Oxidative stress plays a crucial role in LDL oxidation, triggering macrophage uptake and foam cell formation.
How Vitamin E Inhibits Foam Cell Formation
Vitamin E (α-tocopherol) is a potent antioxidant that prevents oxidation of LDL particles .
By reducing oxidative stress , vitamin E lowers the amount of oxidized LDL (oxLDL) available for macrophage uptake , thus inhibiting foam cell formation .
This mechanism may help reduce the progression of atherosclerosis .
Breakdown of Incorrect Options:
Vitamin B12 → Incorrect
Vitamin B12 (cobalamin) is essential for DNA synthesis and red blood cell formation , but it does not directly prevent LDL oxidation or foam cell formation .
Vitamin K → Incorrect
Vitamin K is primarily involved in blood clotting and bone metabolism .
It does not have a major role in inhibiting oxidative stress or LDL oxidation .
Vitamin D → Incorrect
Vitamin D plays a role in calcium homeostasis and immune function , but its direct effect on LDL oxidation and foam cell inhibition is minimal .
Some studies suggest vitamin D deficiency is linked to cardiovascular disease , but it is not a primary antioxidant.
Vitamin B6 → Incorrect
Vitamin B6 (pyridoxine) is involved in amino acid metabolism and neurotransmitter synthesis .
While it may support vascular health , it does not directly inhibit LDL oxidation or foam cell formation .
“Consider which lipoprotein not only transports lipids but actively works to prevent vascular damage. Which one has a role beyond mere transport and acts as a guardian against oxidative stress, inflammation, and cellular death?”
97 / 101
Category:
CVS – BioChemistry
Which lipoprotein particle has anti-oxidant, anti-inflammatory, and anti-apoptotic properties and contributes to inhibiting atherosclerosis?
Lipoproteins are responsible for the transport of lipids in the bloodstream, and different classes of lipoproteins have distinct roles in lipid metabolism and cardiovascular health. Among these, HDL is unique in its protective effects against atherosclerosis , exhibiting anti-oxidant, anti-inflammatory, and anti-apoptotic properties.
Why is HDL the correct answer?
Anti-Oxidant Properties:
HDL helps neutralize oxidative stress by inhibiting the oxidation of low-density lipoproteins (LDL), a key step in the formation of atherosclerotic plaques.
It carries paraoxonase-1 (PON1) , an enzyme that prevents the oxidative modification of lipids in LDL and cell membranes.
Anti-Inflammatory Effects:
HDL reduces vascular inflammation by inhibiting the expression of adhesion molecules (e.g., VCAM-1, ICAM-1) on endothelial cells, which prevents the recruitment of inflammatory cells.
It also helps clear pro-inflammatory oxidized lipids from LDL and tissues.
Anti-Apoptotic Role:
HDL promotes cell survival by activating Akt signaling and inhibiting pro-apoptotic pathways in endothelial and smooth muscle cells, maintaining vascular integrity.
It reduces endothelial dysfunction, a precursor to atherosclerosis.
Role in Atherosclerosis Inhibition:
HDL facilitates reverse cholesterol transport , carrying excess cholesterol from peripheral tissues (including macrophages in atherosclerotic plaques) back to the liver for excretion.
This process helps prevent foam cell formation , a crucial step in plaque development.
Why are the other options incorrect?
Intermediate-Density Lipoproteins (IDL):
IDL is a transitional form between Very Low-Density Lipoproteins (VLDL) and LDL.
It is pro-atherogenic , as it can be taken up by the liver or converted into LDL.
Unlike HDL, IDL does not have significant anti-inflammatory or anti-oxidant properties.
Chylomicron Remnants:
These are leftovers of chylomicrons after triglycerides have been offloaded to tissues.
They are absorbed by the liver but have no known protective cardiovascular effects .
In fact, their accumulation in circulation is associated with increased atherogenic potential.
Chylomicrons:
Chylomicrons transport dietary triglycerides from the intestines to peripheral tissues.
They are not involved in reverse cholesterol transport and do not protect against atherosclerosis.
Their remnants, if not cleared efficiently, can contribute to atherogenic lipid profiles.
Low-Density Lipoproteins (LDL):
LDL is highly atherogenic because it delivers cholesterol to peripheral tissues and can be oxidized into oxidized LDL (oxLDL) , which promotes foam cell formation in atherosclerosis.
Unlike HDL, LDL does not have antioxidant or anti-inflammatory effects; rather, it contributes to endothelial dysfunction and plaque development.
The ascending aorta is the first segment of the aorta, emerging from the left ventricle . It primarily supplies blood to the heart itself . Which arteries branch directly from the ascending aorta before it continues as the aortic arch?
98 / 101
Category:
CVS – Anatomy
Which of the following is a branch of the ascending aorta?
Why “Coronary Arteries” is the Correct Answer?
The right and left coronary arteries arise directly from the ascending aorta , just above the aortic valve at the right and left aortic sinuses (of Valsalva) .
These arteries supply oxygenated blood to the myocardium (heart muscle) .
The right coronary artery (RCA) supplies the right atrium, right ventricle, SA node (in most cases), and parts of the left heart .
The left coronary artery (LCA) branches into the left anterior descending (LAD) artery and circumflex artery , supplying the left ventricle and interventricular septum .
Why the Other Options Are Incorrect?
Esophageal Arteries – Incorrect
The esophageal arteries are branches of the thoracic (descending) aorta , not the ascending aorta.
Mediastinal Arteries – Incorrect
These arteries arise from the thoracic aorta , not the ascending aorta.
They supply tissues within the posterior mediastinum .
Pericardial Arteries – Incorrect
Pericardial branches arise from the thoracic aorta , supplying the pericardium.
The coronary arteries supply the myocardium, not the pericardium .
Posterior Intercostal Arteries – Incorrect
The posterior intercostal arteries originate from the thoracic aorta , supplying the intercostal spaces .
They do not arise from the ascending aorta.
“Consider the patient’s age, symptoms, and physical examination findings. What is the most likely underlying condition, and which investigation would provide the most direct and non-invasive assessment of that condition?”
99 / 101
Category:
CVS – Radiology
An 18-year-old male presents to the outpatient clinic with complaints of palpitations for the last two hours and frequent episodes of joint pain for four years. He has a blood pressure of 110/60 mmHg and a regular pulse of 120 beats per minute. On examination, a mild systolic murmur is heard in the apical area. Which of the following investigations should be advised to the patient?
Why Trans-thoracic Echocardiography (TTE) is Correct:
The patient is an 18-year-old male with palpitations, a history of joint pain, and a mild systolic murmur in the apical area. These findings raise suspicion for rheumatic heart disease (RHD) , a condition that can develop after untreated or recurrent streptococcal infections (e.g., rheumatic fever). RHD often affects the heart valves, particularly the mitral valve, leading to murmurs and palpitations.
Trans-thoracic echocardiography (TTE) is the most appropriate initial investigation because it provides detailed imaging of the heart’s structure and function, including the valves. It can confirm the presence of valvular abnormalities (e.g., mitral regurgitation or stenosis) and assess the severity of the condition. TTE is non-invasive, widely available, and highly effective for diagnosing structural heart disease, which is likely in this case.
Why the Other Options Are Incorrect:
Exercise Tolerance Test:
This test is used to evaluate for coronary artery disease (CAD) or assess exercise capacity in patients with known heart disease. However, this patient is young and has no risk factors for CAD. His symptoms (palpitations and joint pain) and findings (systolic murmur) are more suggestive of valvular heart disease, not ischemic heart disease.
Nuclear Imaging:
Nuclear imaging (e.g., myocardial perfusion scans) is primarily used to assess blood flow to the heart muscle and diagnose ischemic heart disease. This is not relevant to a young patient with valvular heart disease and no symptoms of ischemia.
Coronary Angiography:
Coronary angiography is an invasive procedure used to diagnose coronary artery disease. It is not indicated in this case because the patient’s symptoms and findings are not consistent with CAD. Additionally, coronary angiography is not the first-line investigation for valvular heart disease.
Dobutamine Stress Echocardiography:
This test is used to evaluate for ischemic heart disease or assess myocardial viability in patients with known CAD. It is not appropriate for this patient, as his symptoms and findings point toward valvular heart disease rather than ischemia.
Consider the mechanism leading to impaired heart function due to inadequate blood flow . Which condition specifically results from obstructive changes -> causing reduced oxygen supply to the heart muscle?
100 / 101
Category:
CVS – Pathology
Which of the following terms describes the impairment of heart function due to inadequate blood flow caused by obstructive changes in the coronary circulation to the heart?
The question describes a condition where the heart’s function is impaired due to reduced blood supply caused by obstructions in the coronary circulation . This is the hallmark of coronary heart disease (CHD) , also known as ischemic heart disease (IHD) .
Why “Coronary Heart Disease” is the Correct Answer:
Coronary heart disease (CHD) is caused by atherosclerosis of the coronary arteries , leading to:
Narrowing of blood vessels → Decreased oxygen supply to the myocardium.
Myocardial ischemia → Chest pain (angina), especially with exertion.
Severe obstruction or rupture of a plaque → Myocardial infarction (heart attack).
Chronic ischemia → Progressive weakening of heart function, potentially leading to heart failure.
The underlying pathology is atherosclerosis , where lipid plaques form inside the coronary arteries, reducing blood flow.
Why the Other Options Are Incorrect:
Cerebrovascular Disease – Incorrect
Cerebrovascular disease affects the brain’s blood supply , leading to conditions such as stroke and transient ischemic attacks (TIAs) .
It involves cerebral arteries, not coronary arteries .
Congenital Heart Disease – Incorrect
Congenital heart disease includes structural defects present from birth (e.g., atrial septal defect, ventricular septal defect, Tetralogy of Fallot).
It is not caused by obstructive changes in coronary circulation .
Rheumatic Heart Disease – Incorrect
Rheumatic heart disease is caused by immune-mediated damage to heart valves following a Streptococcus pyogenes infection (rheumatic fever) .
It primarily affects the valves (mitral stenosis, aortic regurgitation) , not the coronary arteries.
Congestive Heart Failure – Incorrect
Congestive heart failure (CHF) is the end-stage result of various heart conditions, including coronary heart disease .
However, CHF itself is a functional outcome of heart failure, not a disease caused directly by coronary artery obstruction.
CHD can lead to CHF , but the question asks about the primary cause, which is CHD .
“Which fetal shunt must close almost immediately after birth to ensure that oxygenated and deoxygenated blood no longer mix in the atria?”
101 / 101
Category:
CVS – Anatomy
The fetal cardiovascular system is designed to serve prenatal needs and permit modifications at birth that establish the neonatal circulatory pattern. Which of the following events is included in the transition from fetal to neonatal circulation?
Understanding the Transition from Fetal to Neonatal Circulation
At birth, the newborn undergoes major cardiovascular changes to transition from fetal circulation (placenta-dependent) to neonatal circulation (lung-dependent). One of the key changes is the closure of the foramen ovale , which occurs due to changes in atrial pressures.
Closure of the Foramen Ovale – A Key Event in Neonatal Circulation:
In fetal life, the foramen ovale allows oxygenated blood from the placenta to bypass the non-functional fetal lungs by shunting blood from the right atrium to the left atrium .
At birth:
The first breath expands the lungs , lowering pulmonary vascular resistance and increasing pulmonary blood flow .
This increases left atrial pressure while right atrial pressure drops due to the loss of placental circulation.
The pressure shift forces the septum primum against the septum secundum , functionally closing the foramen ovale within minutes to hours after birth .
Anatomical closure (forming the fossa ovalis ) occurs over weeks to months .
Breakdown of Incorrect Options:
Vasodilation of Umbilical Arteries → Incorrect
Umbilical arteries constrict, not dilate , after birth due to increased oxygen levels and loss of placental blood flow.
They eventually become the medial umbilical ligaments .
Patent Ductus Arteriosus → Incorrect
The ductus arteriosus closes, not remains patent , as part of the neonatal transition.
Functional closure begins within 12–24 hours , with complete anatomical closure by 1–3 weeks .
Thicker Walls of the Ventricles → Incorrect
Ventricular walls do not immediately thicken at birth ; instead, the left ventricle gradually hypertrophies in response to the increased workload in systemic circulation.
This is a long-term adaptation , not an immediate event in neonatal transition.
Dilatation of the Ductus Venosus Sphincter → Incorrect
The ductus venosus (which shunted oxygenated blood from the placenta to the IVC) closes rather than dilates after birth.
It becomes the ligamentum venosum over the next few weeks.
Your score is
The average score is 68%
Restart quiz
Thank you for your feedback.